CPCE

अब Quizwiz के साथ अपने होमवर्क और परीक्षाओं को एस करें!

A career counselor, who is White, is confronted by an African-American client he has been seeing for several months. The client tells the counselor she feels that he has been pointing her in a career direction based more on her race and gender than on her true abilities. What should the career counselor do first? A. Listen to the client explain why she feels this way, then validate her feelings B. Ask the client to leave, then call an attorney for assistance C. Consult with a supervisor or colleague D. Explain the reasons why this is not true

A

A client says, "I just hate being with my mother. She always needs something from me and it drives me crazy!" In reply, the counselor says, "It sounds like being around her is uncomfortable and frustrating for you." This counselor is using the technique of: A. reflection B. restatement C. congruence D. confrontation

A

A college professor is interested in finding the dispersion of her students' scores on their final course project. What is the best measure this professor could use? A. Standard deviation B. Z-score C. Variance D. Range

A

A counselor has met with a 12-year-old boy with depression for three sessions. Immediately following the third session, the boy's father corners the counselor in the waiting room to ask her what the boy is talking about in the sessions and whether they has figured out why he is so depressed. What is the best way for the counselor to respond? A. "Your son is working hard in therapy, though he and I have agreed that I won't share details with anyone. Maybe next time you could join us at the beginning and your son can decide what he does and doesn't want to share." B. The counselor should ask the boy what he would like to share with his father. C. "Let's set up a phone conversation and I can tell you all about it." D. "I'm sorry, I can't tell you anything about our sessions because they are confidential."

A

A counselor is leading a group for adults who have difficulty with social skills. In the middle of a group discussion about eye contact, one of the group members, Jim, rolls his eyes and turns away from the rest of the group. In this situation, what would be the best response from the counselor? A. "Jim, I see that you just rolled your eyes and turned away from the group. Could you share with us what you're thinking right now?" B. "Jim, it is disrespectful to treat others that way. Please turn back to the group." C. "I think we all saw what just happened here. What do you think Jim is feeling right now?" D. The counselor should say nothing and proceed as if nothing had happened

A

A counselor is leading an eight-member group for survivors of sexual abuse. The counselor asks each person in the group to complete a sociogram. What is the counselor most likely trying to find out by assigning this task? A. How each member perceives his or her position in the group and relationship with other group members B. Whether the group has been effective in helping members overcome past abuse C. What underlying mental health conditions might exist for each member D. How helpful the group members feel the counselor has been as a group leader

A

A counselor is meeting with a man in his thirties for individual counseling. The man initially sought out counseling due to increased anxiety attacks that usually occur on the way to work and during high-stress business meetings. The counselor has been trying to help the man identify underlying emotional processes that occur before these attacks, but the man has difficulty expressing these emotions and seems to feel very uncomfortable whenever the counselor asks him how he feels. Which of the following approaches may be more effective for this client? A. The counselor should focus on the man's cognitive processes and help him establish concrete goals to address B. The counselor should refer the man to a psychiatrist for a medication evaluation to help with his flat affect C. The counselor should confront the man about his inability and reluctance to engage in counseling D. The counselor should help the man explore early childhood experiences that may explain the reasons behind his inability to express feelings

A

A counselor is meeting with a new client for the first session. During the initial interview, the client hesitates and then tells the counselor that he thinks he is bisexual but might not be. What should the counselor do next? A. Ensure the man that she is accepting of all clients, no matter their orientation, and if this is an issue he would like to bring up in counseling, she is receptive to it B. Ask the man why he thinks he is bisexual C. Continue with the interview, and in following sessions bring up the man's comment about possibly being bisexual D. Ask the man if he would be more comfortable meeting with someone who specializes in working with bisexual clients

A

A counselor is performing an assessment on a 22-year-old male client who reports that he has begun hearing voices and feeling as if everyone is out to get him. He says he has difficulty sleeping at night because of these fears and sometimes the voices tell him to do harmful things, such as hurt his children. The counselor performs a risk assessment on the client and does not think that he is at immediate risk for hurting himself or others. What should the counselor do next? A. Refer the client to a psychiatrist for continued evaluation of psychotic symptoms B. Offer to provide supportive, insight-oriented therapy to the client C. Begin educating the client about psychotic symptoms and arrange for him to attend a psychoeducational group D. Invite the client's family members in for family counseling

A

A counselor wants to conduct a research study using surveys to gather information. What is the minimum sample size she should use? A. 100 B. 15 C. 30 D. 75

A

A counselor wants to give a client a vocational aptitude assessment. All of the following would be appropriate measures for this purpose except which one? A. GRE B. ASVAB C. DAT D. GATB

A

A family counselor who consciously joins with the family during sessions, observes what he experiences during those sessions, and then makes interpretations to family members is most likely operating from what perspective? A. Experiential B. Bowen C. Humanistic D. Narrative

A

A group counselor finds out that two members of the group have regularly been spending time together between group sessions to talk about group issues and other members. What should the counselor do? A. Meet with both individuals to explain that group issues should not be discussed outside of group sessions, and suggest that they wait until the group has terminated to continue with their social gatherings B. Ask the group if they would like the individuals to stop socializing outside of the group C. Praise the individuals for forming a meaningful social relationship and encourage them to invite other group members to spend time with them between sessions D. Inform the group members that they have violated the confidentiality of other group members and that they are no longer welcome in the group

A

A group counselor is interested in the long-term relapse rates of individuals in substance abuse groups. After examining relapse rates over a five-year span, the counselor finds that individuals in same-sex groups experienced fewer relapses than individuals in groups with both males and females. This is known as what type of research design? A. Ex post facto B. Descriptive C. Correlational D. True experimental

A

A newly licensed counselor in private practice enjoys working with married couples who are experiencing conflict in their relationships. However, when billing insurance companies for marital therapy, which of the following should the counselor take into consideration? A. Many insurance companies do not reimburse for marital therapy B. Marital therapy is reported to the counseling licensure board to ensure counselors do not commit fraud C. The counselor may not be legally certified to provide marital therapy D. One member of the couple may not agree with marital therapy

A

A student in a college counseling class performs a survey of 100 students at her school. She finds that 8% meet criteria for major depressive disorder. Which of the following statements is true about this student's data? A. There is an 8% prevalence rate of depression in the population surveyed by the student B. There is an 8% incidence rate of depression in the population surveyed by the student C. The prognosis for the students with depression is poor D. The etiology of the students' depression is related to the college they attend

A

A white, Spanish-speaking counselor is meeting with a Hispanic family for family counseling. In order to help this family in the best way possible, the counselor should do which of the following? A. Ask the family questions about their cultural values and beliefs in order to know how to best effect change B. Refer the family to a counselor of Hispanic origin C. Only conduct counseling sessions in Spanish, as that is the family's language of origin D. Treat the family like she would a White family, and assume that the family will let her know if they are uncomfortable with her approach

A

According to Krumboltz, which of the following is most likely to be the task of the career counselor? A. Challenging preconceived notions of acceptable career options B. Helping the individual resolve ego-relevant crises C. Working with the individual to identify a zone of acceptable career alternatives D. Administering assessments to determine which career path fits the individual best

A

After school has been in session for two months, a counselor wants to measure the organization of a second-grade class to identify which children are most popular. Which of the following would be the best approach? A. Use sociometric tools B. Conduct a single-subject design C. Survey the classroom teachers D. Ask students to complete self-report questionnaires

A

All of the following are advantages to receiving counseling in a group setting, except which one? A. There is a lack of norms among group members B. The group setting is a good replication of the participants' everyday world C. Group members can experience social support D. Groups are safe places to practice new skills

A

All of the following are examples of paralanguage, except which one? A. The direction in which a client crosses her legs B. Whether the client speaks fast or slowly C. The silences that occur during a counseling session D. The tone of voice a client uses when she describes her children

A

All of the following are functions of Chi Sigma Iota except which one? A. To provide mediation when an ethical conflict arises B. To contribute to the development of leaders in the counseling field C. To promote research in counseling D. To educate counselors in ways to maintain professionalism in counseling relationships

A

All of the following are true about CACREP exceptwhich one? A. It was founded in 1940 B. It accredits programs at master's and doctoral levels C. It establishes standards for professional competence D. It prepares future practitioners

A

All of the following are true regarding stanine scales except which one? A. They calculate how test scores rank on a scale of 100 B. Scores are always positive C. They are used to convert scores to a single digit D. Scores always range from zero to nine

A

All of the following are true statements regarding older adults except which one? A. Some intellectual decline into a person's late 70s is inevitable B. Depression and other types of mental illness affect 10 percent of older adults C. Physical impairments afflict most older adults by age 60 or 70 D. Most personality traits are stable into late adulthood

A

All of the following are typical goals of multicultural groups except: A. increase group members' understanding of norms acceptable by the majority culture B. help group members understand how new skills can be integrated into the norms of their own cultures C. understand the circumstances that brought group members into the group D. provide learning for the individual in the group process

A

An individual wants to quit smoking, but has found it difficult to do so for an extended period of time. What type of group is the best option to help this individual reach their goal? A. Behavioral counseling B. Rational emotive behavior counseling C. Transactional analysis D. Gestalt theory

A

An individual's score on an assessment has a z-score of 0. What does this mean? A. The individual's score is equal to the mean B. The individual's score is the furthest away from the mean C. The range for the standard deviation is -1.0 to 1.0 D. The individual's score is equal to the range

A

Budgeting, supervision, marketing, and evaluation are all essential components of: A. managing a counseling program B. developing an effective group therapy intervention C. determining what needs a program has D. providing services to individuals and families

A

Choosing a Vocation was published in 1909 and was written by: A. Frank Parsons B. Jesse Davis C. Clifford Beers D. Sigmund Freud

A

Delirium, Parkinson's disease, and Alzheimer's disease are all what type of disorder? A. Neurocognitive B. Neurodevelopmental C. Disruptive D. Stressor-related

A

During a group session, one of the members admits that he sometimes feels relieved that his wife died from cancer several months ago, because they had a difficult marriage. The group member is tearful and obviously upset as he talks, and the counselor responds by validating the group member's feelings in a genuine way. She praises him for opening up to the group and invites other group members to sensitively respond to the disclosure. According to Yalom, this counselor is embodying which of the four leader functions? A. Caring B. Executive leadership functions C. Meaning attribution D. Emotional stimulation

A

Erik Erikson is well known for his stages of psychosocial development from birth to death. Whose explanation of career development parallels these eight stages? A. Tiedeman B. Gottfredson C. Holland D. Super

A

Factorial analysis of variance measures: A. the effects of two or more independent variables on one dependent variable B. the effects of one variable at three or more levels C. the effect of one or more independent variables on a controlled dependent variable D. the effects of two or more independent variables on more than one dependent variable

A

How has an increased pressure to examine the "bottom line" in the field of counseling contributed to intervention and treatment? A. It has led to more research studies on short-term therapies B. It has neglected children and adolescents while focusing only on adults C. It has discouraged counselors from wanting to provide comprehensive, effective mental health treatment D. It has resulted in more wraparound services for individuals with serious mental illnesses

A

In 2015, five states legally permitted physician-assisted suicide. Under what circumstance is this permitted? A. The individual must have a terminal illness B. The individual must have clinical depression and feelings of worthlessness C. The individual's family must agree that the individual's happiness is compromised D. The physician must agree that the individual is prepared to die

A

In his book The Seasons of a Man's Life, Daniel Levinson presents his Stage-Crisis View theory, which examines the typical periods in a man's life. What does Levinson say about midlife crises? A. They are part of normal, healthy development B. They are often the main cause of divorce and other relationship disruptions C. They are social constructs that must be avoided D. They are experienced by every man and woman

A

In most research studies, who should have access to the data? A. The researchers and research assistants only B. Members of the IRB that approved the study C. All of the study participants D. The general public

A

In the 1980s and 1990s, American society saw a drastic increase in the number of people with mental illnesses in homeless shelters and prisons. Though community mental health centers existed, there were not enough resources to serve everyone who needed treatment. This change is attributed to: A. deinstitutionalization B. the stock market crash C. increased numbers of people diagnosed with schizophrenia D. The worsening housing market and lack of support for single parents

A

It has been said that psychoanalytical counseling approaches may appeal more to multicultural clients than other types of approaches, such as Gestalt counseling. All of the following are reasons that could support this claim EXCEPT: A. Multicultural clients typically have traumatic and impressionable childhood experiences B. Multicultural clients may like its emphasis on conceptual relativity C. The formality of the psychoanalytic therapist may appeal to multicultural clients D. The focus on family dynamics may appeal to multicultural clients

A

Of the following behavioral disruptions, family counseling is most often recommended for which one? A. Feeding and eating disorders B. Sexual dysfunctions C. Sleep-wake disorders D. Paraphilic disorders

A

R. K. Coyne developed four levels of group intervention. What are these levels? A. Individual, Interpersonal, Organization, Community B. Initial, Transition, Working, Termination C. Assessment, Planning, Treatment, Evaluation D. Forming, Norming, Storming, Performing

A

The American Counseling Association defines counseling as a profession that helps clients to accomplish what four types of goals? A. Mental health, wellness, education, and career B. Social, cultural, individual, and ethnic C. Individual, family, community, and culture D. Immediate, short-term, long-term, and lifelong

A

The Drama Triangle, developed by Stephen Karpman, is helpful when understanding destructive interactions that occur between individuals in conflict. What three roles are included in the Drama Triangle? A. Victim, persecutor, and rescuer B. Talker, listener, and ignorer C. Ego, superego, and id D. Helper, enabler, and perpetrator

A

The Kruskal-Wallis test is used when: A. you have more than two mean scores on a single variable B. you have scores for two samples and these scores are correlated C. you collect data from two independent samples and the scores are not normally distributed C. you have nominal data that do not fall along a normal curve

A

The parents of a three-year-old child come to see a counselor for help with their son. He has been throwing extreme temper tantrums for the past year to the point that he will yell, scream, and sometimes hit his head on the floor. The parents report that the tantrums didn't start out this bad but have gradually gotten worse. When the child starts having a tantrum, the parents say that they tell him to stop and try to talk to him about his feelings. If this doesn't work, they try to ignore him until he becomes so disruptive that they hold him in a bear hug until he calms down. The parents' unsuccessful intervention is a(n): A. positive-reinforcing stimulus B. example of classical conditioning C. negative-reinforcing stimulus D. punishment

A

There is a general rule that a certain percentage of the population is adequate when determining sample sizes for studies. What is this percentage? A. Five to ten percent B. 25 to 35 percent C. 20 to 25 percent D. Ten to 20 percent

A

There is frequently variability within a distribution of scores. Which of the following is used to describe this variability? A. Standard deviation B. Variance C. Stanine D. Range

A

Though counselors may try to use approaches that enhance client's experiences, at times these approaches may exhibit a lack of understanding of other cultures. This concept, conceptualized by Gilbert Wrenn is known as: A. Cultural encapsulation B. Cultural pluralism C. Cultural norming D. Acculturation

A

What are the four stages of Bergan's behavioral model of consultation? A. Problem identification, problem analysis, plan implementation, problem evaluation B. Contracting, problem identification, feedback and planning, plan implementation C. Known to self, known to others, not known to self, not known to others D. Assessment of client's abilities, modeling approaches, implementing interventions, evaluating counselor

A

What are the three broad areas of development? A. Physical, cognitive, and psychosocial B. Physiological, psychological, and social C. Biological, social, and psychological D. Emotional, spiritual, and cognitive

A

What is a commonly used technique for measuring attitudes or opinions, that typically gives respondents five answer choices such as Always, Often, Sometimes, Almost Never, and Never? A. Likert scale B. Chi-square C. Factor analysis D. Scattergram

A

What is not a time when you would use a nonparametric test? A. When you are working with ratio scale data levels of measurement B. When the scores are abnormally distributed C. When scores do not fall along a normal curve D. When the variance of the sample is different from the variance of the population

A

What is the best way to follow up with a client after career counseling has terminated? A. In whatever way the counselor and client agree upon B. Mail-in surveys and telephone calls C. An in-person meeting with the career counselor D. A group session when several of the counselor's clients discuss their experiences

A

What is the final stage of the racial/cultural identity developmental model? A. Integrative awareness B. Dissonance C. Resistance and immersion D. Introspection

A

What is the main purpose of the AASCB? A. To assist counselors moving from one state to another B. To create a universal definition of the counseling profession C. To provide clear guidelines to counselors regarding ethical practice D. To create counselor training standards for all types of counseling

A

What is the typical range for the standard deviation when calculating a z-score? A. -3.0 to 3.0 B. -4.0 to 4.0 C. -1.0 to 1.0 D. -2.0 to 2.0

A

What is typically regarded as the optimum size for an adult group with no co-leader? A. Eight B. Four C. Six D. Ten

A

What type of leadership function emphasizes the leader's duty to manage the group as a social system? A. Executive leadership B. Caring C. Meaning attribution D. Emotional stimulation

A

What type of research is more valued in the counseling field? A. Qualitative and quantitative are equally valued, as they can both make contributions to the field B. Neither quantitative nor qualitative, because both have severe shortcomings C. Qualitative, because it is based on feelings and judgments D. Quantitative, because it is more accurate and predictable

A

Which of the following are examples of standardized scores that are used to compare different test scores for the same individual? A. Z-score and T-score B. Percentile and T-score C. Z-score and stanine D. Stanine and percentile

A

Which of the following is a change from the DSM-IV-TR to the DSM-5? A. Removal of the multiaxial system B. Removal of V codes C. Addition of a chapter on bipolar disorders D. Addition of a Not Otherwise Specified option

A

Which of the following is not true regarding writing and publication of research? A. Manuscripts should be written according to The Chicago Manual of Style B. Sexist language should be avoided C. All authors of the manuscript should be acknowledged D. A manuscript should be submitted to only one journal at a time

A

Which of the following is the best definition of familism? A. The degree to which a client is attached to and identifies with his or her family B. The process of establishing trust with an entire family system C. The belief that peer relationships are more important than family relationships D. The extent to which a counselor incorporates family interventions into his or her practice

A

Which of the following is true regarding older workers? A. Intellectual functioning is usually not impaired B. Older adults' job satisfaction is lower than younger adults' C. Older adults experience a gradual improvement on speeded tasks D. Older adults' unemployment periods are shorter than younger adults'

A

Which of the following is true? A. Tests can be reliable but not valid B. Valid tests are only reliable if there is a change in an underlying trait or characteristic C. Tests are usually reliable but not valid D. Tests are always valid with enough data

A

Which of the following occupations will likely have the most job openings during the next decade? A. Registered nurses B. College professors C. Postal service clerks D. Locomotive firers

A

Which term applies to the anticipated course of a disorder? A. Prognosis B. Etiology C. Incidence D. Prevalence

A

Which type of group counseling approach typically requires the group leader to be the most active? A. Rational emotive behavior B. Person-centered C. Gestalt D. Existential

A

Who is typically regarded as the "Father of Guidance" and developed the trait-factor approach to career counseling? A. Frank Parsons B. John Krumboltz C. John Holland D. E. G. Williamson

A

A counselor is meeting with a client who struggles with chronic substance abuse and depression. The client regularly shares that he feels left out and expresses his belief that life is unfair. According to Adlerian theory, this client is most likely to be a(n): A. middle child B. twin C. oldest child D. second child

A According to Adler, birth order affects individuals' psychological environments and affects the child's unique experience. The eldest child tends to get a lot of attention, is hard-working and dependable, and fears losing love when a sibling comes along. Middle children see themselves in competition with the older sibling(s), may view life as unfair, and often feel left out. The youngest child often tends to go his or her own way and is influenced by all other siblings. Individuals who are only children have difficulty cooperating, often handle adult interactions well, and want to be the center of attention.

Kübler-Ross identified five stages of behaviors and feelings that dying people experience. Which of the following is not one of these stages? A. Anxiety B. Bargaining C. Anger D. Acceptance

A Elisabeth Kübler-Ross is known for her research and development of the five stages of grief: denial, anger, bargaining, depression, and acceptance.

A student scores 82 on a test. The test's Standard Error of Measurement (SEM) is 4.0. Which of the following is true? A. There is a two out of three chance that the person's score falls between 78 and 86 B. There is a 34% chance that the person's score falls between 78 and 86 C. There is a 95% chance that the person's score falls between 78 and 86 D. It is impossible to predict test scores based on the information given

A If a certain assessment has an SEM of 4.0, you first add and subtract four to the individual score to find the range. Therefore, because the score in this question is 82, there is a two out of three chance that the individual's score falls between 78 and 86.

A researcher wants to better understand the impact of protein intake on elementary-aged children's test scores. He arranges for students at a school in an upper-class area to receive a protein-heavy lunch each day for one month, and he tracks their test scores. These scores are compared to those of students at a school in an impoverished part of town. The students at the second school continued to eat their regular diets without special attention paid to protein. The researcher finds that the test scores, on average, are 15 points higher than test scores at the second school. He concludes that a protein-rich diet contributes to academic success. This study lacks which of the following? A. Internal validity B. A hypothesis C. A single-subject design D. A conclusion

A Internal validity refers to the degree to which external influences have been controlled. A common threat to internal validity is the selection of subjects and the individual differences between the subjects.

Which of the following counselors' approaches is most likely to have been influenced by the work of Savickas? A. A counselor who encourages her clients to be flexible workers and to pursue freelance work when possible B. A counselor who sees himself as an expert who relies heavily on inventories C. A counselor who meets with clients in a group format so individuals can share their ideas with each other D. A counselor who asks the client's family for input about feasible career choices

A Mark Savickas is known for his postmodern approach to career counseling that is based on construction theory, which posits that individuals construct their own reality. Savickas' approach considers the changing nature of the workplace and the move toward flexible work schedules and more freelance employment.

Some counselors use the process of circular questioning, which is when different family members are asked the same questions about the same relationships. This approach is used most often in conjunction with which family therapy approach? A. Milan systemic family therapy B. Narrative family therapy C. Strategic family therapy D. Structural family therapy

A Milan systemic family therapy refers to an approach that views the family as a system that aims to maintain balance.

Find Occupations, Skills Search, and Crosswalk are three components found in which of the following? A. O*NET B. DOT C. SIGI 3 D. OOH

A O*NET is a comprehensive database that categorizes occupations according to worker characteristics, amount of education and training needed, and other factors. The Dictionary of Occupational Titles (DOT) has been largely replaced by O*NET but still offers summaries of about 12,000 occupations. The Occupational Outlook Handbook (OOH) is published every other year and offers information about current salaries and projected trends. The System of Interactive Guidance and Information (SIGI 3) is a top-of-the-line computer-assisted career guidance system that can supplement the assessments and guidance of career counselors.

William Perry is known for developing a scheme for intellectual and ethical development. This scheme includes three categories and nine positions, ending in which of the following? A. Post-commitment, when individuals realize commitment is an ongoing and evolving activity B. Dualism, when there are right and wrong answers C. Subjective knowledge, when there are conflicting answers D. Multiplicity, when individuals must listen to their inner voices rather than society

A Perry explains that there are three categories — dualism, discovery of relativism, and commitment to relativism — and nine positions spread across these categories.

Which of the following is an example of unobtrusive measurement? A. Reviewing a client's existing medical records B. Sending a questionnaire asking about a client's eating habits C. Meeting with a family for an initial therapy session D. Having a diagnostic interview with a young child

A Intrusive methods refer to times when clients know that they are the subject of observation and/or data collection. Unobtrusive methods refer to times when the individuals are unaware that data is being collected, such as when you review a client's existing records.

With what kind of measurement might a researcher use a nonparametric statistic such as a chi-square? A. Nominal B. Ratio C. Ordinal D. Interval

A There are four levels of measurement: nominal, ordinal, interval, and ratio. Nominal data refers to numbers that represent categories or qualities of the variable, such as race, gender, and age. Nonparametric statistical measures, which are often used with descriptive data, should be used with nominal data.

A counselor wants to learn more about a client's unconscious needs and anxieties. Which assessment is most likely to reveal some of this information? A. Rorschach B.Differential Aptitude Tests (D AT) C. Myers-Briggs Type Indicator D. California Psychological Inventory (CPI)

A The Rorschach consists of a collection of abstract ink blot images onto which the individual often projects unconscious desires, urges, and needs.

A college student has known for a while that she wants to be a biologist. She is currently taking classes to determine which area of biology she wants to pursue in her career. This individual is in the process of completing which vocational development task, according to Donald Super? A. Specification B. Stabilization C. Implementation D. Crystallization

A The vocational development tasks are crystallization (forming a vocational goal), specification (moving to a more specific career choice), implementation (entering employment), stabilization (performing a job), and consolidation (achieving status and advancing). This student is performing the task of specification, when individuals move from more general career interests to a specific vocational choice.

What is the main difference between emotion and mood? A. Emotions are temporary, while moods are persistent B. Emotions are less intense than moods C. Emotions are defense mechanisms that individuals use to avoid mood D. Moods usually precede emotions

A.

A career counselor operating from a contextual point of view would most likely work toward which of the following goals? A. Encourage the individual to see themselves as separate from their environment B. Assist the individual in making meaning of their own situation C. Focus on the individual's affective experiences rather than cognitive understanding D. Direct the individual in how to change their environment

B

A client tells their counselor that they think they have a drinking problem, because drinking has started to interfere with their functioning and overall quality of life. Which of the following is a physiological symptom this client might have? A. Becoming violent when drunk B. Sweating and shaky hands C. Inability to think clearly D. Inability to hold down a job

B

A counselor asks their client what day it is, where he is right now, and what his name is. The counselor is gathering information for what part of the mental status exam? A. Thought processes B. Sensorium C. Intellectual functioning D. Affect

B

A counselor at an elementary school would like to start a social skills group for kindergartners, who are typically five and six years old. What is the maximum number of group members the counselor should have in each group? A. Eight B. Four C. Six D. Two

B

A counselor has been meeting for several months with a man whose wife recently died after a terminal illness. During one session, the client states that he has been considering shooting himself in the head to "end the pain" and "to see my wife again." The counselor determines that the man has a high risk of suicide, as he continues to state that he is going to kill himself that night with the gun he has in his house, and he will not take himself to the closest crisis center. What should the counselor do? A. Respect the man's decision and allow him to attempt suicide B. Take steps to obtain an involuntary commitment C. Stay with the client at his home to make sure he is safe D. Keep the man with her until he agrees not to shoot himself

B

A counselor is meeting with an older client, age 80, for the first time. The client lost her husband about 15 years ago and in the past year has begun having symptoms of depression, including sleeplessness and irritability. Which of the following is the most helpful goal the counselor can help the client work toward? A. Examine the origins of the client's depression such as childhood trauma B. Build a positive attitude towards the client's self worth C. Encourage the client to see and find support from her own children D. Work through unresolved grief related to the loss of the client's husband

B

A counselor is meeting with the parents of a two-year-old girl. The parents are worried that their daughter has not yet begun to identify as a girl or boy, and they wonder if they are doing something wrong. The counselor can tell the parents that, in general, most children achieve a sense of being male or female by what age? A. Five B. Three C. 15 months D. Two

B

A counselor regularly administers the Self-Directed Search and the O*Net Interest Profiler. What is the counselor most likely trying to determine? A. Individuals' unconscious anxieties, desires, and urges B. Individuals' skills, values, and/or interests as they pertain to vocations and occupations C. Individuals' knowledge of a specific academic subject D. Individuals' ability to learn new information

B

A large factory is moving out of the country and is planning on terminating employment for most of its workers. As a service to these employees, a career counselor at the company meets individually with each worker to provide career assessments, help them identify possible jobs elsewhere, and assist them in networking and making connections at other companies in the area. This approach is known as: A. career education B. outplacement counseling C. retirement counseling D. hidden job counseling

B

A member of an anger management group describes a recent interaction he had with his boss that led to his getting angry and leaving work. The group counselor directs various members of the group to act out the interaction that was just described to them. Which of the following is this an example of? A. Homogeneity B. Psychodrama C. Altruism D. Preaffiliation

B

A mental health practitioner wants to know more about a client's unconscious drives and emotional functioning. Which of the following psychological assessments might help the practitioner gather this type of information? A. Beck Depression Inventory B. Rorschach C. California Psychological Inventory D. WISC-IV

B

A research study recruits only individuals with extremely low social skills on a certain measure. After employing an intervention, the researchers find that many of the study subjects scored at or close to the mean. Which of the following is the most likely explanation for these results? A. Placebo effect B. Statistical regression C. Demand characteristics D. Reactivity

B

A researcher wants to study the relationship between educational level and discipline styles of parents. Which of the following is a null hypothesis? A. Disciplinary styles depend more on the parent's age and gender than on educational level B. There is no correlation between a parent's educational level and how he or she disciplines children C. It is difficult to know how educational levels influence disciplinary styles D. Parents with higher levels of education are less likely to physically discipline their children

B

A school counselor wants to know whether parents of students believe a recent character-building intervention was effective. She sends a survey home with each student and receives 40% of the surveys back. This type of research design is: A. comparative B. non-experimental C. qualitative D. experimental

B

A school-aged child is a member of several systems: family, school, neighborhood, and peers. Who would argue that it is important to examine all systems impacting this child? A. Perry B. Bronfenbrenner C. Levinson D. Bandura

B

A two-year-old boy shouts, "Car!" any time he sees a bicycle, truck, or anything else with wheels. According to Piaget, this is an example of a(n): A. hallucination B. schema C. defense D. misinterpretation

B

According to Corey, what activities are included in the first stage of a group? A. Establishing cohesion and trust B. Recruiting and screening C. Preparing for the outside world and deciding what courses of action to take D. Challenging the leader and experiencing conflict

B

According to Hanna and Cardona, counselors working cross-culturally should use a variety of techniques to help the client achieve freedom from what? A. Childhood trauma B. Ongoing oppression C. The idea that a strong self-concept is important D. Negative family dynamics

B

According to object relations theory, normal progression through four broad stages during the first three years of life is necessary for healthy functioning. All of the following disorders except which one may occur if this normal progression does not take place? A. Narcissistic personality disorder B. Schizophrenia C. Reactive attachment disorder D. Borderline personality disorder

B

According to the ACA Code of Ethics, which of the following is not unethical? A. A counselor and his client agree that they are mutually attracted to each other and use the time in sessions to have sexual contact B. A counselor has sexual contact with a client she saw briefly for therapy ten years earlier C. A counselor agrees to provide couples' therapy to an ex-boyfriend she split from three years ago D. A counselor has sexual contact with one of her supervisees

B

Albert Bandura is credited with the development of social learning theory, which stresses the importance of both social and cognitive factors for individuals. What is one of the central concepts of social learning theory? A. Diversity B. Self-efficacy C. Relativism D. Defense mechanisms

B

All of the following are essential to counseling program planning except which one? A. A mission statement or general philosophy of the program B. A pilot study C. Development of goals D. A needs assessment

B

All of the following are true about non-experimental survey designs except: A. they can measure attitudes, perceptions, and many other subjective experiences B. surveys are time-consuming and difficult to administer C. the response rate of survey research is often below 50 percent D. it is difficult to generalize findings from survey designs

B

All of the following are true regarding Aaron Beck's theory of depression except: A. Internal communication of depressed people is negatively focused B. Depression usually stems from early childhood trauma C. The cause of depression can be a combination of genetic and environmental factors D. Cognitive therapy helps more to alleviate depression than antidepressant medication

B

An individual who has strong social skills, is well-respected, has empathy for others, and understands nonverbal cues is said to possess: A. paradoxical knowledge B. high emotional intelligence C. poor cognitive abilities D. propinquity

B

As counselors are beginning to work with multicultural clients, they may need to do which of the following in order to develop an effective therapeutic relationship? A. Emphasize the meaning of confidentiality and understand that multicultural clients are more sensitive than white clients to counselors speaking with family members or religious leaders B. Share more personal information with multicultural clients than they might with white clients C. Be much more formal with multicultural clients than they would be with white clients D. Be sensitive to the amount of small talk that occurs between counselor and client, as multicultural clients will think this approach is unprofessional

B

By pairing a negative stimuli with positive events, a counselor successfully helps a client overcome his anxiety. This process is known as: A. paradoxical intention B. systematic desensitization C. token economy D. aversion therapy

B

Career counselors have the option of giving many types of assessments to help clients choose careers that match their interests and abilities. The ACT and the ITBS measure: A. Personality B. Achievement C. Interest D. Aptitude

B

Carl Jung's body of work is most closely linked to what personality assessment? A. Thematic Apperception Test B. Myers-Briggs Type Inventory C. Minnesota Multiphasic Personality Inventory (MMPI-2) D. Rorschach inkblot test

B

Clients with dissociative disorders along with depression, anxiety, and/or substance use are at highest risk for: A. personality disorders B. self-injurious and suicidal behavior C. divorce D. psychotic symptoms

B

Don Dinkmeyer's Systematic Training for Effective Parenting (STEP) program integrates concepts used by which group approach? A. Rational emotive behavior B. Adlerian C. Gestalt D. Existential

B

How many divisions of the American Counseling Association are currently in existence? A. 100 B. 18 C. 50 D. 80

B

In a group for individuals who have recently lost a family member to cancer, the counselor notices that one member, an Asian woman, rarely speaks. When she is asked a question directly, she quickly answers but does not elaborate. Which of the following is the most likely reason for her behavior? A. The woman does not understand what is being said by others in the group B. The woman is following cultural norms that discourage sharing personal information with others C. The woman is resisting talking about her late husband because it is too painful D. The woman does not think that the group is helpful to her

B

In order to be effective, group counselors should have certain skills that allow them to work well with group members. All of the following are important core skills counselors should have except which one? A. Managing the group within time constraints B. Knowing the best ways to work with cultures of all types C. Preventing members from blocking productive discussion D. Helping the group to set appropriate goals

B

In what way is family counseling typically different than individual counseling? A. Family counseling aims to create conflict, whereas individual counseling aims to create resolution B. The course of treatment of family counseling is usually shorter in duration than that of individual counseling C. Family counseling views the locus of pathology within one family member, whereas individual counseling views the locus of control in the client's surroundings D. Family counseling requires more extensive training than individual counseling

B

Many different factors influence development, including social, biological, and physical factors. How do parents, in particular, most influence children's development? A. Introducing them to mass media and violence B. Through attachment and disciplinary styles C. Serving as examples through play D. Through exposure to religious beliefs

B

Multidimensional models of abnormal human behavior typically look at what four dimensions? A. Thought processes, appearance, mood, and intellectual functioning B. Biology, cognition, emotions, and culture C. Spirituality, reactivity, passivity, and locus of control D. Adaptation, functioning, etiology, and prognosis

B

Postmodern approaches to therapy are often based on the belief that multiple realities are formed through interactions with others. Which of the following is a postmodern counseling approach? A. Psychodynamic therapy B. Narrative therapy C. Cognitive behavior therapy D. Feminist therapy

B

Psychologist Charles Spearman is best known for which of the following? A. Developing a short-term treatment for psychosis B. Bringing statistical analysis to testing C. Writing the first achievement test for children D. Creating an effective group therapy model

B

Relabeling, therapeutic double binds, and prescribing the symptom are concepts used in: A. group therapy B. structural family therapy C. Bowenian family therapy D. strategic family therapy

B

Sociometry, the study of interpersonal relationships within a group, was first developed by what psychotherapist? A. Irvin Yalom B. Jacob Moreno C. Alfred Adler D. R. K. Coyne

B

Some individuals who have experienced trauma, losses, and other potentially damaging events seem to cope well and function relatively normally in society. What quality do these individuals possess? A. Plasticity B. Resiliency C. Tabula rasa D. Thought blocking

B

Sometimes certain characteristics of study subjects elicit preferential feelings and responses from researchers, altering results. This is known as: A. statistical regression B. experimenter bias C. attrition D. instrumentation error

B

The first professional counseling association, the National Vocational Guidance Association, was founded in what year? A. 1890 B. 1913 C. 1927 D. 1942

B

To determine internal consistency on an instrument with non-dichotomous items, what statistical measure should be applied? A. Standard error of measurement B. Cronbach alpha coefficient C. Coefficient of nondetermination D. Kuder-Richardson formula

B

Under what circumstances are the mean, median, and mode all identical? A. When all scores are skewed in either direction B. When the distribution of scores is symmetrical C. When the scores have a positive skew D. When all scores are evenly divided by the same numbers

B

Under what conditions may a counselor release test results? A. To the test taker's family members B. To competent professionals and with the test taker's consent C. To other counselors who may be interested in the results D. To the counselor's supervisors and program administrators

B

What are the typically recommended treatments for bipolar disorders? A. Group therapy and family counseling B. Mood-stabilizing medication and psychotherapy C. Relaxation training and antipsychotic medication D. Interpersonal therapy and cognitive behavior therapy

B

What does a needs assessment measure? A. The difference between where a program started and where it is now B. The difference between what is and what is desired C. The discrepancies between different individual players in a counseling program D. The similarities between a particular program and other programs like it

B

What is an ideal length of time for outpatient counseling group sessions with adults? A. Two and a half hours B. 90 minutes C. One hour D. 30 minutes

B

What is one advantage of using non-random or nonprobability samples? A.It accurately represents proportions of individuals that exist in the population B. They can yield very useful data C. It is always cheaper than using random sampling D. Results can be generalized to the greater population

B

When a correlation coefficient (r) is 1.00, which of the following is true? A. The mean of all the scores is equivalent to the mode B. If you know one score, you can predict the next score with 100 percent certainty C. Out of 100 scores, half of them will be equal D. There is a 10 percent chance the next score will be the same

B

When providing an interpretation of assessment or test results to a client, what is the most important thing a counselor can do? A. Provide the client with a written report B. Ask the client about his or her feelings and thoughts in response to hearing the results C. Provide recommendations for treatment based on test results D. Invite the client's friends and family members in order to increase the client's support

B

When research for a study is gathered over a long period of time, what threat to internal validity is most likely? A. Experimenter bias B. Maturation C. Statistical regression D. Hawthorne effect

B

Which of the following children is most likely to suffer from maltreatment? A. The youngest child in a two-parent household who was recently diagnosed with anxiety B. An elementary-aged child of a single parent living in a poor neighborhood C. A toddler living with their grandmother and grandfather D. A middle-class African-American child whose ADHD symptoms are managed with medication

B

Which of the following distinguished Roe's approach to career development from that of others? A. Roe operated from a social learning approach in which career selection is heavily influenced by social experiences B. Roe believed that the parent-child relationship was a central determinant in career selection C. Roe focused on self-efficacy and social cognitive theory in her explanation of career selection D. Roe took a developmental approach to vocation

B

Which of the following is considered a test of personality? A. Self Directed Search (SDS) B. Myers-Briggs Type Indicator (MBTI) C. Minnesota Importance Questionnaire D. O*Net Interest Profiler

B

Which of the following is false about the phenomenon of group cohesion? A. Group members are more likely to identify with members that are perceived as powerful B. Attacks on a cohesive group result in less cohesion C. Attacks on a cohesive group do not necessarily result in a change in group cohesion D. Attacks on a cohesive group result in greater cohesion

B

Which of the following is true regarding contextualism? A. It uses psychodynamic techniques such as interpretation B. It is based on the idea that an individual cannot be separated from his or her environment C. It focuses on how societal norms influence individuals' experiences D. It is the belief that individuals make decisions based on their cognitive, rather than emotional, experiences

B

Who was a developmental psychologist known for the belief that children learn best through interactions with others? A. Erik Erikson B. Jean Piaget C. John Bowlby D. Sigmund Freud

B

A graduate student earning his counseling degree wants to conduct a research study examining the relationship between educational level and substance abuse. What type of research would be most helpful in this situation? A. Experimental comparative design B. Non-experimental correlational design C. Non-experimental survey design D. Quasi-experimental design

B A non-experimental correlational design uses a correlation coefficient to describe the relationship between variables. The student in this question could gather information about individuals' current substance use and level of education, then calculate the relationship between the two without using any type of experimental design.

Circumscription and Compromise focuses on which of the following? A. A person's career as an extension of his or her personality B. Career preferences as they develop in childhood C. The role of social learning in career selection D. The fact that people choose careers to address unmet needs

B According to Gottfredson, vocational self-concept develops in childhood and influences occupation selection.

In rational emotive behavior therapy (REBT), what does the "A" in A-B-C-D-E mean? A. Accept B. Action C. Affect D. Approach

B Action (A) that occurs externally from the individual; a Belief (B) that is a form of self-verbalization; a Consequent affect (C) that can be rational or irrational; Disputing (D) the irrational belief that leads to the behavior; and an Effect (E), the change in the self-verbalization.

What is the resulting ego virtue for the final stage of Erikson's eight psychosocial stages? A. Hope B. Wisdom C. Love D. Will

B During the final stage, integrity versus despair, individuals look back on their lives and either view them as meaningful or have regrets. The resulting ego virtue is wisdom. The resulting ego virtue for intimacy versus isolation (early adulthood) is love. The resulting ego virtue for autonomy versus shame and doubt (ages one and a half to three) is will. The resulting ego virtue for trust versus mistrust (birth to one and a half) is hope.

All of the following are criticisms of Kübler-Ross's stages except which one? A. The stages do not take into consideration the fact that feelings can be recurring B. The stages negate the importance of the person feeling angry about their illness C. The stages do not always occur in the same linear fashion D. The stages do not adequately represent the complexity of the dying person's feelings

B Elisabeth Kübler-Ross is known for her research and development of the five stages of grief: denial, anger, bargaining, depression, and acceptance.

In what type of situation is autocratic leadership the best option? A. A closed group that does not allow changes to membership B. A group that only meets for four sessions C. A group made up of motivated and self-directed members D. A group whose members have many differing ideas

B Group counselors can choose from one of three main styles of leading groups: autocratic, democratic, and laissez-faire. Autocratic leaders are typically direct and set clear goals and boundaries for groups. This type of leadership works best when time restraints are an issue, when membership frequently changes, or when the group needs to coordinate with other groups. Democratic leaders allow members to express their ideas and aim to encourage discussion and participation. Laissez-faire leaders allow group members to be in charge of all decision-making and it works best when group members are very motivated.

According to John Holland, career choice is an expression of what? A. Unmet needs B. Personality C. Society D. The economy

B He identified six personality types that develop due to parental influences, genetic factors, and the environment. These types include realistic, investigative, artistic, social, enterprising, and conventional.

Within a set of scores, the mean is 102, the median is 116, and the mode is 120. What is most likely about this distribution of scores? A. Its skew is unknown B. It has a negative skew C. Its skew could be positive or negative depending on the number of scores D. It has a positive skew

B In this question, because the mode of 120 and the median of 116 are higher than the mean of 102, the curve has a negative skew.

Some mental health clinicians, called Neo-Freudians, continue to practice psychoanalysis but have made some modifications to Freud's original theories. Which of the following is true of Neo-Freudians? A. They discount psychodynamic and sociodynamic forces B. They place more emphasis on the ego than the id C. They believe that childhood experiences are not as important as present interactions D. They eliminate the concept of unconscious drives

B Neo-Freudians are psychoanalysts who have moved away from Freud's emphasis on the id as the main psychological force. More emphasis is placed on the ego, which is controlled by the reality principle.

Chi-square and Mann-Whitney U Test are examples of what type of statistics? A. Inferential B. Nonparametric C. Parametric D. Descriptive

B Nonparametric statistics, such as chi-square and the Mann-Whitney U test, are used when data is not normally distributed and variances are inconsistent. Parametric statistics, such as the t-test and analysis of variance, can be used when samples are randomly drawn from the population and results are distributed along a normal curve.

Which of the following is the best example of self-efficacy? A. A college senior takes out a personal loan to start her own company and loses all the money in the first year B. A high school graduate with mediocre grades enrolls in community college to become a nursing assistant because he believes he is capable of performing this particular career C. A law school student drops out and chooses to become a paralegal assistant because she wants to begin working right away D. A psychiatrist becomes trained as a clinical social worker because he feels he needs social work skills in order to best help clients

B Self-efficacy, an individual's belief that he or she can perform some necessary task when he embraces his abilities he is displaying self-efficacy

Which of the following is an example of a directional hypothesis? A. There is no difference between the rates of girls and boys who attend college after high school B. More girls who graduate from high school will attend a four-year college than males who graduate from the same high schools C. It is impossible to measure the differences between girls and boys who attend college after graduating from high school D. The proportion of girls and boys who graduate from high school and enroll in a four-year college will be different

B The null hypothesis states that there are no effects of the independent variable on the dependent variable and therefore no differences in the control and test groups. Directional hypotheses state that the scores of one particular group will be significantly different than scores in another identified group. Nondirectional hypotheses state that there will be statistical differences between groups, but it is unclear which group's scores will be higher or lower.

Steve de Shazer used the analogy of a skeleton key to describe how he helped clients develop interventions that they could use to solve many different problems. What type of philosophy did de Shazer use in his therapeutic approach? A. Cognitive behavior B. Social constructionist C. Narrative D. Feminist

B The social constructionist perspective operates from the viewpoint that we use language to construct a common reality with others, and that there are no objective "functional" family dynamics that apply to all family systems.

A couple that currently has a three-year-old daughter is deciding whether to have another child. Based on Alfred Adler's concept of birth order, what changes might this couple expect to see in their daughter if they do have a new baby? A. The older child will likely be overprotected by parents and have a hard time sharing B. The older child will likely become more dependent on the father C. The older child will likely become rebellious and competitive D. The older child will likely have an easygoing personality

B following the birth of a second child the older sibling will likely seek comfort from the father. The oldest child can become perfectionist and authoritative, yet can learn to bear responsibility and be helpful to others.

What is the relationship between the first and second stages of the career counseling process? A. The second stage, providing information, is only important if the first stage, assessment, determines more information is needed B. The first stage, establishing a relationship, helps progression to the second stage, developing a better understanding of the client's problems C. The second stage, assessment, cannot be completed without successful completion of the first stage, providing information D. The first and second stages, establishing a relationship and assessment, are interchangeable

B several steps: establishing a relationship, identifying the problem, conducting an assessment, providing information, making a decision, and implementing the plan.

A mother is angry with her teenage daughter for breaking curfew. When the daughter finally comes home, the mother yells, "You're ungrateful and self-centered, and I can't wait until you're out of this house for good!" The mother then retreats into her bedroom and doesn't speak with her daughter until the daughter approaches her mother to ask for a ride to school the next day. The mother drives her to school but ignores her the entire time. This is called: A. triangulation B. emotional cutoff C. ethnocide D. differentiation

B Emotional cutoff occurs when an individual deals with conflict by reducing contact with the other person.

When sharing her moral beliefs with her counselor, a client states that she believes there are universal ethical principles, but she wouldn't be afraid to go against these principles in the name of human rights such as justice or liberty. What stage of Kohlberg's moral development is this client in? A. Conventional B. Postconventional C. Preconventional D. Self-interested

B Lawrence Kohlberg identified three levels of moral development that relate to the relationship between the individual and society. In the first level, preconventional, individuals judge the morality of an action based on its immediate consequences and rewards. During the conventional level of moral reasoning, individuals judge morality based on the comparison of actions to society's expectations. During the final level, postconventional, individuals realize they are separate from society and can have principles and ethics different from the greater society.

A researcher wants to determine the impact that income level, educational achievements, race, and religious values have on whether children graduate from college. What statistical measure would give the best predictive power of these variables on the dependent variable? A. Scatterplot B. Multiple regression C. Factor analysis D. T-test

B Multiple regression can be used when a researcher wants to examine the strength of the relationship of independent variables on a dependent variable.

When would a researcher use nonparametric (rather than parametric) statistics? A. When it can be assumed that the sample is homogeneous to the variance of the population B. When no assumption about the variance of the population scores can be made C. When data is normally distributed along a bell-shaped curve D. When a sample is randomly drawn from a population

B Parametric statistics, such as the t-test and analysis of variance, can be used when samples are randomly drawn from the population and results are distributed along a normal curve. Nonparametric statistics, such as chi-square and the Mann-Whitney U test, are used when data is not normally distributed and variances are inconsistent.

Five students scored as follows on an exam: 74, 80, 81, 91, and 97. What is the inclusive range? A. 23 B. 24 C. 81 D. 84.6

B The range is the highest score minus the lowest score. The inclusive range, which includes every individual score, is the highest score minus the lowest score plus one (1). 97 - 74 = 23 + 1 = 24

An African-American counselor has recently noticed that he feels increasingly irritated after sessions with a particular client, a white middle-aged man. After consulting with a colleague, the counselor realizes that he feels resentful toward the client because of his own resentment toward white individuals in general for oppression and discrimination against minorities. The counselor is experiencing what phenomenon during his sessions with this client? A. Personalism B. Countertransference C. Acculturation D. Transference

B countertransference, in which the counselor experiences feelings toward the client(s

A career counselor has met with a transgender client for several sessions. Standardized assessments have suggested that the client's strengths lie in teaching, and the client tells the career counselor that she is thinking of becoming an elementary school teacher. What is the best way for the counselor to discuss the probability that the client will encounter discrimination in her future career? A. The counselor should refer the client to an individual therapist who can help the client when she is discriminated against in the teaching field B. The counselor should carefully point the client in the direction of a career that is more accepting of transgender employees C. The counselor should gently inform the client that discrimination by some employers, particularly in teaching and caretaking professions, can be very strong and they should suggest that the client feel supported by friends and family as she plans to pursue this career D. The counselor should ignore the fact that the client is transgender, as it is illegal to bring this up in career counseling

C

A career counselor is meeting with a client for the first time. This client, a middle-aged Asian woman, wants to speak with the counselor about her desire to get a part-time job now that her children have left for college. Which of the following is true as the counselor begins working with this client? A. The counselor should be careful not to address race, since this might offend the client B. The counselor should suggest only opportunities that he knows are typically pursued by Asian women C. The counselor should ask questions about the client's personal values, particularly as they pertain to cultural norms D. The counselor should offer to speak with the client's husband due to the patriarchal nature of many Asian families

C

A company with at least 50 employees is legally required to allow all of the following individuals (each of whom has worked full-time for the company for at least one year) to take 12 weeks of leave, except which one? A. A single female employee adopts a five-year-old and wants to take two months off work to help the child get settled B. A married father wishes to take 12 weeks of leave beginning the day his new son is born C. A young employee, adopted at birth, just found her biological parents and wants to take leave to spend time with them D. A middle-aged employee's mother is terminally ill and the employee needs to care for her during the last weeks of her life

C

A counselor asks a group of one hundred high schoolers to complete a self report questionnaire about their drug use and sexual activity. After recording the results of the questionnaire, the counselor strongly believes that the students consistently underreported this behavior. What factor MOST likely influenced the students' answers? A. Placebo effect B. Self-fulfilling prophecy C. Social desirability D. Groupthink

C

A counselor employed at a shelter for women who are victims of aggressive sexual assault tells her supervisor that she has begun having nightmares recently about being raped. She also says that when she works late and leaves after dark she feels panicky, her heart races, and she feels like she is being watched. She has been having trouble focusing during sessions with women in the shelter and often feels trapped in her own office when the women are talking about being abused. This counselor is most likely suffering from: A. Transference B. Compassion fatigue C. Secondary traumatic stress D. A normal part of the therapeutic process.

C

A counselor following Salvador Minuchin's approach to family therapy would most likely focus on which of the following? A. Metacommunication in families B. Connections between family members that keep the system in balance C. Transaction patterns in the family D. Differentiation of self among family members

C

A counselor has been providing play therapy to young children for many years. She has recently noticed that many of the parents of the children she works with have had questions about the therapeutic orientation of play therapy, why play therapy is helpful, and how it works. What type of group might this counselor find to be most helpful for the parents of these children? A. A counseling group for parents to work through resentment and anger related to having a child in counseling B. A self-help group to alleviate the stress felt due to parenting children with emotional problems C. A psychoeducational group providing information about play therapy D. A psychotherapy group for parents to begin to see their own contributions to their children's illnesses

C

A counselor in a small town is contacted by the parents of a young boy who is experiencing anxiety at school. The family is Vietnamese and the counselor is concerned that she is not familiar enough with Vietnamese culture to help the family. What should the counselor do first? A. Read as much as possible about Vietnamese culture B. See the child for an assessment, and decide after the assessment whether she can continue seeing him C. Address her concerns with the family D. Tell the family she cannot work with them, but avoid telling them why

C

A counselor is conducting diagnostic sessions with a family and wants to learn more about the family's dynamics, unspoken rules, and communication patterns. Which of the following would be the best tool for the counselor to use? A. Open system B. Ecomap C. Genogram D. Strategic approach

C

A counselor is meeting with a family for the first time to perform an assessment. During the session, the counselor collects information about the family's problem and each family member's opinion on the issue. The counselor also notices which family members speak more than others and what facial expressions they use when others are speaking. Finally, the counselor observes where family members choose to sit, which is also known as what? A. Triangulation B. Nonverbal communication C. Proxemics D. Kinesics

C

A counselor operating from a reality therapy perspective is recruiting clients for a group for recovering alcoholics who continue to have interpersonal difficulties. Which of the following should group members be prepared to do? A. Restructure cognitions in preparation for change B. Explore childhood experiences and family constellations C. Take responsibility for their behaviors and formulate a plan for change D. Learn how to modify behaviors through modeling

C

A counselor specializing in family therapy wants to ensure they are gender-sensitive when providing services. In order to do this, the counselor should do all of the following except which one? A. Identify the strengths and needs of both men and women in the family B. Recognize their own preconceptions and beliefs about gender roles in families C. Avoid encouraging families to analyze their beliefs about gender roles D. Help the family identify the roles of men and women in families

C

A counselor tells a client that the client is only going to be as successful as they believe they can be. This is an example of: A. constructivist theory B. social learning theory C. self-efficacy theory D. psychosocial theory

C

A counselor working from the perspective of feminist psychology is least likely to believe which of the following? A. Clients are experts on themselves B. Therapy is meant to advance a transformation in society as well as in clients C. Clients should be examined outside of their sociocultural environments D. Symptoms are often types of survival strategies

C

A counselor working with a four-year-old child gives the child a small piece of candy after he completes three tasks in a row. This is an example of what type of reinforcement schedule? A. Variable interval B. Fixed interval C. Fixed ratio D. Variable ratio

C

According to most career counselors, what is the firststep in the career counseling process? A. Assess the client's strengths using inventories and standardized instruments B. Provide information to the client about various career options C. Establish a relationship with the client D. Identify the client's problem

C

All of the following are acceptable rationales for using tests EXCEPT: A. to identify interests not previously known B. to help the client gain self-understanding C. to establish a baseline to submit to a health insurance company D. to help the client predict future performance in training or work

C

All of the following are principles underlying existential theory except which one? A. With freedom of choice comes personal responsibility B. Clients have the freedom to choose what they do and how they react C. Self-talk is the source of emotional disturbance D. Individuals are motivated to find meaning in their life journeys

C

All of the following are recommended assessment resources except which one? A. Mental Measurements Yearbook B. Tests in Print VIII C. Diagnostic and Statistical Manual - 5th edition D. A Counselor's Guide to Career Assessment Instruments

C

All of the following are true about the White identity development model except: A. whites are exposed to racial beliefs and attitudes B. the final stage of the White identity development model is integrative awareness, from which a non-racist identity emerges C. although Whites experience societal forces, they do not often experience introspection or dissonance D. the stages of the identity development model are the same for Whites and for minorities

C

All of the following are true regarding co-leadership of a group except which one? A. If possible, one co-leader should be male and the other female B. Co-leaders can serve as models and recreate situations for the group C. Co-leaders should engage in power struggles from time to time to demonstrate conflict resolution skills D. Co-leaders should give each other feedback

C

All of the following are true regarding social justice counseling except: A. it seeks a balance of power and resources B. it addresses issues of unequal power C. it has not grown as a specialty in the past 25 years D. it has led to ACA identifying advocacy competencies for counselors

C

All of the following are true regarding women who seek counseling except which one? A. Women are more susceptible to depression than men B. Women are usually more emotionally expressive than men C. Women are more likely to successfully complete counseling than men D. Women are more likely to seek counseling than men

C

All of the following violate the rights of a group member except which one? A. The counselor tells a group member that unless he wants his family to know about his infidelity, he'd better keep coming to the group B. A group member becomes emotionally overwhelmed, and the counselor prevents him from exiting C. The counselor requests that a group member who has been monopolizing the discussion wait and let others speak D. When group members opt to discuss community agencies that they have found helpful, the counselor comments that the group is not for talking about resources but for discussing feelings

C

An elementary school student is given a test of cognitive ability. He scores in the 80th percentile as compared with other students his age and grade level. What does this student's score mean? A. When the student takes the test again, he has an 80% chance of earning the same score B. The student has a raw score of 80 on the test and has an above-average IQ C. The student's score is higher than 79% of the scores, and 20% of the scores are higher than his score D. The student scored higher than 80% of the scores, and 19% of the scores are higher than his score

C

An elementary school teacher expects that the African-American boys in her class will perform below average, so she automatically places all of these students in the lowest reading and math groups. The teacher also expects that the boys will require more behavioral interventions and disciplinary actions than the White students. Sure enough, by the end of the first quarter, all of the African-American boys are underperforming and have been sent to the principal's office more than any other group of students in the classroom. This is an example of what common phenomenon? A. Attribution theory B. Integrative awareness C. Self-fulfilling prophecy D. Identity development

C

Carl Jung is known for his belief in the collective unconscious. His theory serves as a way of understanding humans' emotional responses to issues of self and purpose. Which of the following is based on Jung's theory? A. Beck Depression Inventory B. Rorschach Inkblot Test C. Myers-Briggs Type Indicator D. WISC-IV

C

Compassion fatigue is common in the counseling field. Which of the following is not a symptom of compassion fatigue? A. Boredom during sessions B. Loss of empathy C. An increased desire to help clients D. Loss of interest in client concerns

C

Disruptive mood dysregulation disorder and premenstrual dysphoric disorder are two diagnoses new to the DSM-5. Under which DSM-5 category do both of these diagnoses fall? A. Bipolar and related disorders B. Anxiety disorders C. Depressive disorders D. Obsessive-compulsive and related disorders

C

During a group counseling session about sexuality issues, group members discuss the differences between male and female stereotypes. One group member comments that she disagrees with some of these stereotypes. Several of the other members quickly begin to criticize her statements and make rude comments about her appearance and intelligence, requiring the counselor to intervene. These group members were: A. blocking B. linking C. scapegoating D. intellectualizing

C

During a session with a client, a counselor notices that the client has not bathed in several days and looks unkempt. The client makes statements such as "it would be better if I weren't around" and tells the counselor that they would use a gun that is now in their house to kill herself if they felt hopeless enough. When the counselor asks the client whether they might end their own life soon, the client replies that they aren't sure. What is the best option for the counselor at this time? A. Call the client's close family members and friends so they can provide 24/7 observation B. Give the client her personal number and instruct the client to call counselor when and if they decide to commit suicide C. Inform the client of the counselor's duty to protect them, and advise them to go to the closest crisis center or hospital D. Call law enforcement so they can take the client to be evaluated

C

Each state has different licensure laws for counselors. Due to this fragmentation, what is one current threat to counselors' licensure? A. Some states may give excessive leniency to counselors in regard to administering psychological tests B. Unless a counselor is certified, he or she cannot practice in more than one state at the same time C. The scope of counselors' practice may become increasingly narrow D. There is a high possibility that licenses can be revoked for minor ethical infractions

C

Historically, intellectual assessments consistently found differences between cultural and racial groups. What is the best explanation for these differences? A. Cognitive assessments were rarely given to minorities B. Men were smarter and better educated than women C. Assessment instruments were targeted to favor Whites D. Blacks were less sophisticated and savvy than other racial groups

C

Human growth and development can be conceptualized as either qualitative or quantitative in nature. Which of the following is an example of quantitative change? A. Erikson's stages of psychosocial development B. Progression of self-concept over time C. An improved score on a depression screening D. Maslow's hierarchy of needs

C

In 1976, Gail Sheehy wrote Passages: Predictable Crises of Adult Life. What is one of the main points of this work? A. Positive relationships are essential in order for individuals to navigate stages successfully B. Transitional times are when negative life events tend to occur C. Transitions from one life stage to another are opportunities for growth D. Everyone goes through the same transitional periods

C

In the Archway model, one of the pillars represents individual characteristics. What does the second pillar represent? A. The individual's career options B. The person's socioeconomic status, education, and intelligence C. Family, the labor market, and other external factors D. Religious beliefs and cultural influences

C

In the adolescent years and later, what factor is most important when determining which occupation is selected? A. Gender roles B. Intelligence level C. Self-awareness D. Social valuation

C

Information-processing theories and behavioral theories fall into what broad category explaining how humans grow and develop? A. Cognitive theories B. Psychoanalytic theories C. Humanistic theories D. Learning theories

C

John Crites' model of career counseling includes three factors when diagnosing the career problem. What are these three components? A. Dynamic, motivational, and synthesis B. Analysis, collection, and synthesis C. Differential, dynamic, and decisional D. Synthesis, prognosis, and follow-up

C

Judith Jordan and others are credited with developing the self-in-relation theory. The principal components of this theory are used to support what general topic? A. Neo-Freudian theory B. Cultural identity C. Women's development D. Social learning theory

C

Masters and Johnson are widely known for their contributions to the field of sex therapy. According to Masters and Johnson, which of the following best describes the cause of sexual dysfunction? A. The biological aging process prohibits enjoyable sexual experiences B. Society discourages freedom and disinhibition during sex C. The participant is too critical of his or her own performance D. The individual's partner has unreasonable expectations

C

One major change in the DSM-5 is the reliance on: A. ICD-10 descriptors B. a multiaxial system C. dimensional assessments D. therapy and counseling treatments

C

Over the past 50 years, there has been a push for accountability in the counseling field, primarily motivated by: A. client empowerment B. new psychiatric medications C. funding D. serious mental illnesses

C

Quasi-experimental design is similar to true experimental design with the EXCEPTION of which of the following? A. Quasi-experimental designs measure the existence of relationships between two or more variables, while true experimental designs measure the degree of the relationship B. Quasi-experimental designs descrive a state of events, while true experimental designs determine cause and effect relationships. C. Quasi-experimental designs do not randomly assign subjects to treatment and control groups, while true experimental designs do use randomization. D. Quasi-experimental designs investigate whether there are differences between two groups, while true experimental designs explore why these differences exist.

C

Sandra Bem's Sex-Role Inventory found which of the following? A. 25% of all adolescents have had sexual experiences with members of the same sex B. About half of all college students do not use birth control methods on a regular basis C. About 30% of children and young adults are androgynous D. Only about 40% of young adults view themselves as in control of their own sexual experiences

C

Stanford-Binet Intelligence Scales and the Wechsler Adult Intelligence Scale (WAIS-IV) both measure all of the following except: A. the ability to adapt to the environment B. cognitive ability C. the effects of learning on an individual D. the ability to think in abstract terms

C

The "performing" stage of group development refers to which of the following? A. A stage when conflict is inevitable as members work out personality differences B. A stage when group members establish goals for the group but continue to focus on themselves C. A stage when members trust each other and often achieve success D. A stage when the group assesses the work they have accomplished

C

The Individuals with Disabilities Education Act of 2004 (IDEA) provides for supplemental funds for what group of individuals? A. Minority individuals over the age of 18 B. Adults 65 and older C. Children from birth through age two D. Young adults ages 18 to 21

C

The National Counselor Exam is an example of what type of assessment? A. Ipsatively interpreted B. Speed-based C. Criterion-referenced D. Norm-referenced

C

The National Defense Education Act was passed in 1958 and did which of the following? A. Allowed any individual to attend public school no matter their age B. Enforced discrimination in public schools C. Provided money for the training of school counselors D. Increased spending for schools with military programs

C

The normal curve, also called the bell-shaped curve, distributes scores into six equal parts. What percentage of scores include two standard deviations? A. 50% B. 68% C. 95% D. 99%

C

The stimulus variable is also known as the: A. deviation from the mean B. dependent variable C. independent variable D. hypothesis

C

There are broad categories of individuals who have unique or special needs, such as religious or racial classifications, women, gays and lesbians, poor people, and children. This is known as: A. cultural encapsulation B. disproportionality C. cultural pluralism D. prejudice

C

There are four steps in the planning of a counseling program. What are these four steps, in order? A. Conceptualization, establishment, development, evaluation B. Modification, implementation, development, evaluation C. Conceptualization, development, implementation, evaluation D. Development, operation, evaluation, termination

C

Typically, instruments that have reliability coefficients of what value or higher are said to be reliable? A. .30 B. .50 C. .70 D. .90

C

What conflict is likely to arise if a counselor uses cognitive behavioral approaches with multicultural clients? A. Multicultural clients are likely to not want the therapist to take an active approach B. Multicultural clients will likely misunderstand the counselor due to the language barrier C. The counselor's solutions may not be consistent with multicultural clients' beliefs D. Multicultural clients may want a short-term, solution-oriented approach

C

What group has the highest rate of unemployment in the United States? A. Hispanic females B. Hispanic males C. Black males D. White females

C

What is an example of face validity? A. A test that predicts outcomes accurately B. A test that measures a phenomenon accurately C. A test that looks valid D. A test that covers certain content accurately

C

What is the main problem with early career theories? A. They were developed mostly in Europe rather than America B. They often reference data from samples too large to analyze C. They tend to be limited to white, college-educated males D. They are generalizable only to lower-class workers

C

What is the most accurate definition of convergent validation? A. The result of comparing multiple traits to one dependent variable B. The result of no correlation between the identified construct and others C. The result of a high correlation between the identified construct and others D. The result of a test accurately predicting future scores

C

What is the purpose of establishing a feedback loop throughout the planning of a counseling program? A. To help establish goals and objectives for the intervention B. To assess various areas of need in the program C. To make sure development is proceeding appropriately D. To provide a system of checks and balances to show insurance companies that their money is well-spent

C

What psychologist published Counseling and Psychotherapy in 1942? A. Irvin Yalom B. Gilbert Wrenn C. Carl Rogers D. Sigmund Freud

C

What type of assessment is most often included as part of a job application? A. Personality B. Achievement C. Aptitude D. Intelligence

C

When there is a discrepancy between a nonverbal and a verbal message, which of the following is true? A. The individual sending the messages is aware of the differences between the two B. The speaker is intentionally avoiding the truth C. The nonverbal message will be believed and the verbal message will be minimized D. The verbal message will be more convincing than the nonverbal behaviors

C

Which of the following are considered the most effective psychotherapeutic interventions for depressive disorders? A. Cognitive behavior therapy and Gestalt therapy B. Cognitive behavior therapy and family therapy C. Cognitive behavior therapy and interpersonal therapy D. Cognitive behavior therapy and Reality therapy

C

Which of the following can be used to see how reliable a test would have been had it not been divided? A. Duncan's new multiple range test B. Scheffe's test C. Spearman-Brown formula D. Kuder-Richardson formula

C

Which of the following is a possible threat to external validity, but not to internal validity? A. Placebo effect B. Experimenter bias C. Ecological validity D. Selection of subjects

C

Which of the following is a topic commonly addressed in group couples counseling? A. De-escalating crisis situations B. Addressing the effects of childhood trauma on relationships C. Learning effective parenting skills D. Reducing the amount of physical violence in the home

C

Which of the following is an accurate statement about deductive research? A. It tends to be descriptive and correlational B. It leads to the building of a specific theory C. It uses existing theories to explore relationships between certain elements D. It is practical in nature

C

Which of the following is an example of a class-bound value? A. A counselor places emphasis on verbal communication during sessions B. A counselor encourages her clients to be emotionally expressive C. A counselor expects clients to adhere to weekly 50-minute sessions D. A counselor focuses her interventions on the individual client

C

Which of the following is an example of a unit that might be used in cluster sampling? A. All female students enrolled in a statewide university system B. Children whose parents have divorced in the past 10 years C. All residents living in one city block D. Because 25% of the population in a city is Hispanic, 25% of the study subjects are Hispanic as well

C

Which of the following is false regarding family counseling? A. Family counseling leads to stronger and healthier families B. Family counselors believe that the power of relationships can heal and stabilize the family unit C. Nuclear families make the best family counseling clients D. Family counseling aims to increase communication between family members

C

Which of the following types of children is least likely to be abused or neglected? A. Defiant B. Premature C. Overweight D. Hyperactive

C

Which therapeutic approach was developed by Eric Berne and operates from the belief that a life script develops in childhood and influences a person's behavior throughout adulthood? A. Gestalt therapy B. Psychoanalysis C. Transactional analysis D. Existential therapy

C

Who created the term "eugenics" and was one of the first individuals to study intelligence testing? A. Robert Williams B. Alfred Binet C. Sir Francis Galton D. David Wechsler

C

Why might meaning attribution be an important function of group leaders? A. It can help group members feel validated and accepted B. It may create an environment in which group members feel more comfortable expressing deep emotions and personal values and beliefs C. It helps group members have a common understanding of what is being expressed, and can increase familiarity with emotions and feelings D. It provides a structure and framework for each group session, implementing predictability and consistency

C

Why might the t-test be used? A. To evaluate more than one dependent variable B. To control the influence of one or more independent variables on the dependent variable C. To determine whether the mean scores of two groups are significantly different from each other D. To determine the likelihood that the null hypothesis is true

C

A client says, "I never knew what to expect from my father growing up. Sometimes he would explode and beat me and my sisters for no reason, and other times he would be the most loving dad in the world." According to Carkhuff, what would be a Level 1 response to this statement? A. "You look like this bothers you for some reason." B. "You look very worried when thinking about your dad's unpredictability." C. "Why do you think he did that?" D. "Your uncertainty about your dad's reactions to you and your siblings has contributed to your anxiety today and why you might feel so afraid of getting into a serious relationship with someone."

C "Why do you think he did that?" is a Level 1 response, as it fails to address the client's affect and focuses only on the content by asking a blunt question about the client's father's behavior. "You look like this bothers you for some reason" is a Level 2 response because it subtracts from the client's affect. "You look very worried when thinking about your dad's unpredictability" is a Level 3 response, as it can be interchangeable with the client's affect and content. The best response would be, "Your uncertainty about your dad's reactions to you and your siblings has contributed to your anxiety today and why you might feel so afraid of getting into a serious relationship with someone." This is a Level 4 or 5 response as it adds to the client's affect and meanings.

Effective group leaders typically use what level of emotional stimulation? A. Very low amounts B. Moderately low amounts C. Moderate amounts D. High amounts

C According to Yalom, effective leaders use moderate amounts of emotional stimulation, moderate amounts of executive direction, frequent use of caring functions, and consistent use of meaning attribution.

Taylor is a young boy who speaks in three-word sentences, such as "I want cookie." He often refers to any day in the past as "yesterday," even if the event occurred weeks before. He has an active imagination and has recently developed a fear of monsters in his closet. According to Piaget's theory of development, what stage of development is Taylor in? A. Formal operational B. Concrete operational C. Preoperational D. Sensorimotor

C During the preoperational stage, which typically occurs between ages two and seven, the child begins to speak in multi-word sentences, expands his or her imagination, engages in symbolic play, and begins to possess a relative sense of time. The sensorimotor stage occurs during the first two years of life, when children experience the world through crawling and beginning to walk, as well as with their five senses. The concrete operational stage occurs from age seven to eleven and is marked by an increased ability to think logically and about others' perspectives. The formal operational stage is the fourth and final stage and occurs from age eleven through adulthood. During this stage, children develop abstract thought and complex problem-solving skills.

What role do psychodynamic techniques play in Crites' model of career counseling? A. They are used as part of the assessment and inventory process B. Crites does not recommended psychodynamic techniques due to their long-term nature C. They are used after the problem is diagnosed to help the client interpret and understand why the problem was occurring D. They are used before diagnosis takes place so the client can begin to formulate reasons why they are having problems

C John Crites' model of career counseling is a comprehensive approach that requires the counselor to diagnose the career problem using differential, dynamic, and decisional approaches. Crites also believed that once the career problem has been diagnosed, the counselor should provide client-centered counseling, psychodynamic techniques, and trait-factor and behavioral approaches.

A career counselor is interested in using a computer guidance system with numerous assessments to measure clients' interests, values, and skills. Which of the following is the most comprehensive system? A. O*NET B. CHOICES C. SIGI 3 D. Focus II

C O*NET is a comprehensive database that categorizes occupations according to worker characteristics, amount of education and training needed, and other factors. Focus II and CHOICES are both computer-assisted career guidance systems, but they have limited assessment components.

A client tells their career counselor that they are interested in having an annual salary of about $50,000, and they want to know what types of careers are currently on the rise. They have a bachelor's degree and are not interested in going back to school. What tool might the career counselor use to help this client narrow down their search? A. DOT B. SIGI 3 C. OOH World-of-Work map

C OOH) is published every other year and offers information about current salaries and projected trends. The World-of-Work map is a tool developed by ACT that organizes occupations based on primary tasks related to People, Data, Things, and Ideas. The Dictionary of Occupational Titles (DOT) offers summaries of about 12,000 occupations. The System of Interactive Guidance and Information (SIGI 3) is a top-of-the-line computer-assisted career guidance system that can supplement the assessments and guidance of career counselors.

A counselor operates from the viewpoint that clients' difficulties often result from problems with interpersonal relationships that originate in the first three years of life. What theory best matches this counselor's approach? A. Transactional analysis B. Person-centered theory C. Object relations theory D. Gestalt theory

C Object relations theory is based on psychoanalytic concepts, including the belief that early childhood is incredibly relevant to how clients operate within relationships in the present day. Object relations theories teach that, in the first three years of life, infants have the opportunity to develop significant relationships with primary caretakers that will shape their interactions with people throughout their lives.

When a researcher wants to measure one variable at three or more levels (such as level of income), what statistical measure would be most appropriate? A. ANCOVA B. MANOVA C. One-way ANOVA D. Factorial ANOVA

C One-way Analysis of Variance (ANOVA) can be used to determine differences between three different forms of one variable, such as level of income. Multivariate Analysis of Variance (MANOVA) is the statistic used when there is more than one dependent variable involved in the analysis. Analysis of Covariance (ANCOVA) is used to determine covariance when an independent variable and its impact on dependent variables is controlled.

Which of the following is the best example of a primary group? A. A group for individuals whose spouses have recently died B. A group for sexual abuse perpetrators run by former victims of child abuse C. A psychoeducational group for teenagers who are at risk of developing depression D. A support group for women in the military who have just returned from combat

C Primary groups emphasize preventing problems and developing healthy behaviors. Secondary groups focus on reducing the severity of a specific problem and include preventative and remedial elements, such as adjusting to a loss. Tertiary groups involve rehabilitation and focus on returning individuals to healthy functioning.

A 20-year-old is in counseling for severe sexual and physical trauma she experienced when she was four and five years old. The client decides that she is ready to talk about the trauma with the counselor, but when details of the trauma come up during sessions, the client takes on the voice of a little girl, curls up in a ball on the floor, and acts immaturely. What is the defense mechanism this client is using to cope? A. Repression B. Reaction formation C. Regression D. Introjection

C Regression is the return to a much younger psychological and emotional state due to overwhelming feelings. Reaction formation occurs when someone expresses the opposite feeling of what he or she may actually feel. Introjection is the process of using fantasy to identify the expression of an impulse. Repression is the act of forgetting or denying an idea that creates anxiety or other uncomfortable feelings.

High internal consistency means that a measure is: A. dichotomous B. valid C. reliable D. obtrusive

C Reliability refers to the degree to which a study can reproduce results. Internal consistency is the degree to which a test measures what it is intended to measure, while also producing the same results each time.

All of the following are commonly used post hoc tests except which one? A. Newman-Keuls test B. Scheffe's test C. Mann-Whitney U test D. Tukey's HSD test

C Researchers might apply a test after the analysis of variance is calculated (post hoc) if it is unclear as to which mean scores are significantly different from each other.

A woman with schizophrenia has an individual therapist she meets with once a week and a psychiatrist she sees every other month for medication. What is the best type of group for this individual? A. Secondary B. Support C. Tertiary D. Primary

C Tertiary groups are best for those with more serious types of pathology, such as severe depression or psychosis. Many participants in tertiary groups also see an individual therapist on a regular basis. Primary groups are more focused on healthy living and coping strategies and include support and psychoeducational groups. Secondary groups aim to reduce or prevent the severity of existing problems.

Which of the following is an example of a tested interest? A. A client's former jobs consisted of being a hostess at a restaurant, a salesperson at a bookstore, and a summer camp counselor B. In his free time, a client enjoys running, reading, and gardening C. According to an assessment, a client's strengths are a match with the social work profession D. A client's college major was art history

C Tested interests are those measured by assessments or tests, such as an inventory that shows how a client's strengths match those needed in the social work profession

After having mostly white friends since she was in school, an African-American teenager begins to develop friendships with other African-Americans. She and her new friends often challenge assumptions and endorsements of their primarily white classmates, resulting in an increased sense of identity and appreciation of her race. What stage of the racial identity development model is this individual experiencing? A. Dissonance B. Conformity C. Resistance and immersion D. Introspection

C There are five stages of development for individuals in the minority race or culture. The first stage, conformity, is characterized by identification with the majority. The second stage, dissonance, is when the individual's self-concept is challenged and there is a conflict between appreciation and depreciation of self. The third stage, resistance and immersion, is when the individual rejects the majority and accepts the minority views. The fourth stage, introspection, follows the intense emotions of the resistance and immersion stage and is more focused on self-appreciation. The final stage, integrative awareness, occurs when the individual can appreciate aspects of both the dominant and minority cultures.

Which of the following would not be characteristic of a Transactional Analysis Group? A. Content including dynamic ego states B. Leader-centered C. A focus on A-B-C theory D. A focus on being free of scripts and games

C Transactional Analysis Groups focus on the study and repair of the pattern of one's interpersonal interactions, with a focus on being free of scripts and games. Transactional Analysis includes content based on dynamic ego states (parent, adult, and child), and is leader-centered in its character.

In what type of qualitative research is observer bias particularly important? A. Historical analysis B. Legal analysis C. Ethnography D. Case study

C Ethnography is a type of interactive research in which the researcher collects data through interviews and observations about a group or system.

Which of the following would not be characteristic of a Transactional Analysis Group? A. Content including dynamic ego states B. Leader-centered C A focus on A-B-C theory D. A focus on being free of scripts and games

C Transactional Analysis Groups focus on the study and repair of the pattern of one's interpersonal interactions, with a focus on being free of scripts and games.

YAVIS and QUOID are acronyms used to describe what types of clients? A. Cognitive and emotional B. Similar and dissimilar C. Desirable and undesirable D. Insured and uninsured

C YAVIS refers to desirable clients and stands for Young, Attractive, Verbal, Intelligent, and Successful. QUOID stands for Quiet, Ugly, Old, Indigent, and Dissimilar.

A counselor has been meeting with a client, Jim, for over three years at the same time every week. The counselor is looking for new clients to add to his practice and receives a call from a potential client who would like to come in during the same time slot as Jim. The counselor tells the new client that he is free at that time and thinks, "Jim will understand. I'll just ask him to come at a different time for a while." This is a violation of all of the following ethical principles except which one? A. Veracity B. Justice C. Autonomy D. Fidelity

C There are many principles that underlie counselors' ethical decision making. These include beneficence, the desire to work for the good of the client as well as for society; nonmaleficence, avoiding harm; autonomy, respecting the client's right to self-determination; justice, treating individuals fairly; fidelity, honoring commitments; and veracity, being honest and truthful. By prioritizing a new client over another client, this counselor is violating the principles of fidelity, justice, and veracity. Ethically, the counselor should reserve the appointment time for the existing client and attempt to find another time to meet with the new one.

Which of the following disorders is typically considered to be more ego-syntonic than ego-dystonic? A. Bipolar I disorder B. Obsessive-compulsive disorder C. Anorexia nervosa D. Panic disorder

C Ego-dystonic behaviors are those that the individual considers unacceptable.

Which of the following is an acceptable objective of a counseling program? A. Counselors in the student counseling department will build relationships with each other in order to build a greater sense of community and trust B. The counseling department will attend suicide risk trainings this year to better assess suicidality in the student body C. By the end of the program, students' in-school suspensions will decrease from four per semester per student to two per semester D. Students will experience improved self-esteem over the course of the counseling group

C -specific

"Generation X" refers to individuals born between 1965 and 1976. Which of the following characterizes this group? A. Preferring traditional marriage roles and linear career paths B. Goal-oriented and civic-minded C. Multiculturally inclusive and self-confident D. Wanting exciting jobs and keeping options open

D

"Positive Uncertainty" is a model of career decision-making associated most closely with whom? A. Frank Parsons B. John Crites C. Mark Savickas D. H B Gelatt

D

A 4 year old child regularly becomes upset when his mother is not able to read him a bedtime story. He frequently makes comments like "I wish Daddy was dead." and often tells his father to go in another room so he and his mother can be alone. What is MOST likely true regarding this child? A. It is likely that he is being abused by his father B. He and his mother are enmeshed C. He is showing early signs of mental illness. D. He is experiencing a typical Oedipal complex.

D

A Solomon four-group design is used in which of the following circumstances? A. When you have scores for two samples and the scores are correlated B. When you want to determine the strength of the relationship of independent variables on a criterion variable C. When you have nominal data and want to determine whether two distributions differ significantly D. When you want to determine the effect of a pretest

D

A career counselor has a positive relationship with a client they have known for several months. However, after identifying the client's problems, conducting assessments, and helping the client obtain additional information, the client is still unable to commit to making a decision about her career. What should the career counselor do next? A. Refer the client to another career counselor who may have a different, more useful, approach B. Suggest the client make a decision anyway and see if it works C. Terminate services with the client, as the counselor has reached beyond their scope D. Work with the client to determine the underlying causes of their inability to make a decision

D

A child, age 17, recently dropped out of school and is living at home. He does not have a job, though he sometimes goes with his mother to the animal shelter to volunteer. When a family counselor asks the boy what his career and educational goals are, he shrugs his shoulders while his mother jumps in to explain that he wants to be a police officer but is now "just taking a break." When asked about the boy's responsibilities around the house, the boy's father rolls his eyes and tells the counselor that his wife does everything for their child, including making his meals. The mother tells the counselor that she and her son have a "special" relationship and seems pleased when she says that her husband is not a part of it. This mother and son are most likely: A. Differentiated B. Disengaged C. Resilient D. Enmeshed

D

A counseling student in a non-CACREP program plans to graduate next May. Which of the following is true? A. She is only eligible for national certification by NBCC B. She must pass the National Counselor Exam before graduation in order to become certified C. As long as she logs 1,000 hours of internship experience, she can become certified after graduation D. Before she can become certified, she must have 3,000 hours of post-graduate work experience

D

A counselor facilitating a group for older adults with anxiety and depression has noticed that recently one of the longtime group members, Samantha, has been reluctant to participate. Samantha frequently denies having any recent feelings of anxiety or depression when asked directly, though Samantha's individual therapist has contacted the group counselor to let her know that she is worried Samantha's depression is getting worse. What is the best way for the group counselor to address Samantha's resistance? A. "As a group, let's tell Samantha what we think about her depression lately." B. "Samantha, I just don't understand why you don't want to talk to us anymore. It doesn't make any sense, especially since your depression is getting worse." C. The counselor should not say anything to Samantha; she will begin to participate again when she is ready. D. "Samantha, I've noticed that you've not been as talkative lately, but you seem sad. I wonder if the group could help you figure out why talking about anxiety and depression has been so difficult lately."

D

A counselor has been asked to meet with a 22-year-old African-American male referred to her by his probation officer. Before meeting with the client, the counselor assumes that he will have a negative attitude, will speak disrespectfully, and has been charged with assault or drug possession. The fact that the counselor has preconceived judgments without sufficient knowledge is known as: A. Discrimination B. Pluralism C. Acculturation D. Prejudice

D

A counselor is interested in running more groups for clients who suffer from depression. Which of the following would be the best example of a goal this type of group might have? A. Explore group members' childhoods for answers to current depressive states B. Learn how to talk with family members about what it is like to have depression C. Decrease suicidal thoughts and self-harming behaviors D. Learn how to interact with others in ways that relieve depressive symptoms

D

A counselor is meeting with a 12-year-old boy who she believes would benefit from adventure-based counseling. What outcomes might be expected for the client with this type of counseling? A. More clarity about what diagnosis the client might have B. Better understanding of what aspects of socialization the boy struggles with C. Better awareness of how early childhood experiences have affected his relationships D. Increased self-confidence and communication skills

D

A counselor meeting with a young woman struggling with anxiety suggests that they work together to identify dysfunctional thoughts. The counselor then asks questions regarding the validity and helpfulness of those thoughts and helps the client replace those thoughts with healthy, more reasonable thoughts in order to decrease the client's anxiety. This is an example of what type of counseling? A. Multimodal therapy B. Existential counseling C. Feminist therapy D. Cognitive behavioral counseling

D

A counselor who specializes in women's issues frequently encounters clients who find themselves exhausted from taking care of others, such as children and other family members. The counselor encourages her clients to find ways to establish a better balance between commitment to others and self-care. This concept of healthier balance is explored in which of the following? A. Carol Gilligan's In a Different Voice B. Jean Baker Miller's Toward a New Psychology of Women C. Gail Sheehy's Passages D. Harriet Lerner's The Dance of Intimacy

D

A counselor working at a college regularly provides group therapy for students. The counselor is curious to know whether a particular curriculum produces improved social skills for students with high-functioning autism. The counselor explains to group participants that she is collecting data on their scores through self-evaluations completed by them. After the final data is collected, the counselor sees that participants' scores on the self-evaluations improved drastically over the course of the intervention. However, the counselor is concerned that the scores were heavily influenced by participants' knowledge that the counselor was conducting research. This counselor is appropriately concerned about what threat to external validity? A. Attrition B. Placebo effect C. Experimenter bias D. Hawthorne effect

D

A displaced homemaker meets with a career counselor to get help finding a job now that her children have entered adolescence. Which of the following is most likely true regarding this client? A. Once she gets a job, she will relegate most of the housework and childcare to her husband or partner B. She will need to develop a new skill set, as her former job skills are now obsolete C. She will be unable to retire at the same time as her husband or partner D. She will need help building her job-seeking skills and self-concept

D

A family counselor has been meeting with a married couple and their three children for several months. One day, the father shows up alone for a session and tells the counselor that he is having an affair with a coworker. He has not informed his wife and asks the counselor not to tell her. What should the counselor do? A. Tell the man that if he doesn't tell his wife about the affair before the next session, the counselor will tell her instead B. Because information disclosed in session is confidential, agree to keep the secret from the rest of the man's family C. Inform the man that if he continues with the affair she will not agree to see the family again D. Remind the man that the counselor has obligations to the entire family, and work with him on finding a way to talk with his wife about the affair

D

A fifth-grader is referred to the school counselor because he became upset in class and stated that he wished he weren't alive anymore. What should the counselor do first? A. Speak with the child's parents B. Take the child to the nearest emergency room C. Arrange a meeting between the teacher and the student to discuss the incident D. Perform a risk assessment

D

A group counselor often uses the empty chair technique and guides group members to fantasize about how they might create change in their lives. This counselor is operating from what counseling approach? A. Rational emotive behavior B. Reality C. Transactional analysis D. Gestalt

D

A group counselor tends to act more like a member than a leader and offers suggestions to group members, but is overall non-directive. What type of group is this counselor most likely a part of? A. Adlerian B. Transactional analysis C. Psychoanalytic D. Person-centered

D

A marriage and family counselor has been meeting with a couple for several months. The couple clearly states that they both value loyalty and honesty more than any other qualities in a marriage. In the next session, the husband appears nervous and jittery. Halfway through the session, he tearfully tells his wife that he had sexual contact with another woman the previous week. The discomfort the husband feels is known as: A. Intrinsic motivation B. Familism C. Attribution theory D. Cognitive dissonance

D

A marriage and family counselor has been meeting with a couple for several months. The couple clearly states that they both value loyalty and honesty more than any other qualities in a marriage. In the next session, the husband appears nervous and jittery. Halfway through the session, he tearfully tells his wife that he had sexual contact with another woman the previous week. The discomfort the husband feels is known as: A. intrinsic motivation B. familism C. attribution theory D. cognitive dissonance

D

A married couple with two school-aged children gets divorced, which drastically reduces the amount of conflict in the home. All of a sudden, however, the younger child starts throwing temper tantrums, whereas before he was perfectly well-behaved. What phenomenon might explain this child's change in behavior following the divorce? A. Enmeshment B. Cybernetics C. Permeability D. Homeostasis

D

A researcher in a college setting knows that 55% of the student body is female and 45% is male. For his study on the effects of caffeine on students' study habits, he randomly selects the sample to be 55% female and 45% male. This researcher is using what type of sampling? A. Stratified sampling B. Cluster sampling C. Purposeful sampling D. Proportional stratified sampling

D

A researcher wants to examine the reliability of an instrument by administering it twice on the same group. About how long should the researcher wait between administrations? A. One month B. Three days C. Two months D. Two weeks

D

A significance level for a particular study is .05. What does this mean in terms of the results of the study? A. Data reported in the study has only a five percent chance of being accurate B. If the study is conducted 100 times, researchers are 5% certain their hypothesis will be correct C. Researchers believe that five percent of the time study participants report inaccurate data D. If the study is conducted 100 times, researchers are willing to accept the possibility of rejecting the null hypothesis in error five of those times

D

A student earns a grade-equivalent score of seven on an achievement test. What does this score most likely mean? A. The student is in the 70th percentile of all students who took the same test B. Seven out of ten students perform as well as this student did on the test C. The student has a 70 percent chance of earning the same score if he or she retook the test D. The student got the same number of items correct on the test that the average seventh grader gets

D

A woman in couples' counseling complains that her husband keeps his emotions to himself and refuses to talk about his feelings. After hearing this, the husband turns to his wife in surprise and says, "I try to tell you how I feel, but you're the one who never wants to talk about your emotions!" The woman is most likely using which of the following defenses? A. Repression B. Denial C. Reaction formation D. Projection

D

A young girl decides she wants to be a doctor when she grows up, but after reading several children's books about doctors she tells her mother she is going to be a nurse "because doctors are boys and nurses are girls." In what stage of vocational development is this child, according to Gottfredson? A. Orientation to gender allowances B. Orientation to size and power C. Orientation to social valuation D. Orientation to sex roles

D

After having a group discussion, group members agree on a less conservative course of action than they would have taken individually. This is known as: A. blocking B. triangulation C. linking D. risky shift phenomenon

D

After meeting with a client several times, a career counselor realizes that the client is struggling with indecisiveness that pervades not only his career decisions but his personal life as well. What should the counselor recommend to the client? A. Continued assessments and inventories until career choices can be narrowed down B. Continued career counseling with a different provider C. Trial-and-error approach in which the client tries various careers to find which one he likes most D. Personal counseling to resolve emotional and psychological issues

D

All of the following are possible threats to internal validity except which one? A. Experimenter bias B. Statistical regression C. Instrumentation D. Ecological validity

D

All of the following are stages of a group as defined by Irvin Yalom, except which one? A. Orientation B. Termination C. Cohesion D. Storming

D

All of the following are true regarding CACREP exceptwhich one? A. The ACA is focusing on CACREP as a unifying force B. It exerts a considerable amount of influence over the counseling profession C. Academic institutions are moving to require CACREP program graduation for admission D. It was established in 2010

D

All of the following describe qualitative research except: A. studies individual units in naturalistic settings B. assumes that there are multiple realities C. researchers may use their impressions, judgments, and feelings D. researchers examine for causes and relationships

D

Appraisal is an important concept in the field of assessment. Which of the following is the best definition of appraisal? A. Making a conclusion about the usefulness of measurement data B. Collecting information about human behavior C. Determining the dimensions of a specific attribute or trait D. Making judgements about human behavior

D

Arthur Jensen believed that what factor was the most powerful indicator of intelligence? A. Personality traits B. Environment C. Socioeconomic status D. Genetics

D

Assertiveness training promotes all of the following except which one? A. An ability to express positive and negative emotion B. The ability to stand up for one's rights while respecting others' rights C. Understanding the differences between assertiveness and aggression D. Justifying one's own actions in relation to others' behaviors

D

Based on data from the United States census, the number of Americans with varying degrees of disability is increasing. What is the most likely reason for this increase? A. Higher numbers of more dangerous jobs B. Society's more reckless attitude in general C. Fewer employees covered by health insurance D. The increasing older population

D

Beginning in the 1950s and '60s, Donald Super conceptualized career development to include specific stages and tasks. Which of the following is a common criticism of Super's early ideas? A. The stages and tasks lacked clear explanations and definitions B. They neglected the role of self-concept in careers C. They did not address the retirement stage of peoples' lives D. They applied mostly to white, middle-class, college-educated males

D

Career counselors often assess the ways individuals use their free time. Which of the following theories is commonly used to determine what types of leisure activities people engage in? A. Present vs. Future B. Personal vs. Social C. Internal vs. External D. Compensatory vs. Spillover

D

Computer programmer and chemist are examples of what type of career, according to Holland? A. Conventional B. Enterprising C. Realistic D. Investigative

D

Fantasy, tentative, and realistic are three stages of career development presented by: A. Miller-Tiedeman B. John Holland C. Ann Roe D. The Ginzberg Group

D

Formal education, observational learning experiences, and diverse interpersonal interactions form which of the following? A. Gestalt therapy B. Psychoanalysis C. Rational emotive behavior therapy D. Humanistic life outlook

D

Gelatt's decision-making process consists of what five steps? A. Connect, motivate, maintain, terminate, evaluate B. Interview, decide, maintain, motivate, terminate C. Assess, de-escalate, intervene, stabilize, evaluate D. Recognize, collect data, examine outcomes, attend to values, evaluate and decide

D

How did the Tarasoff case in 1976 impact counselors' duty to warn? A. It ruled that any citizen, not just counselors, must inform law enforcement if a threat is made toward someone else B. It suggested that unless counselors are completely sure of a client's intent, they do not need to warn potential victims C. It required that counselors inform law enforcement when they are seeing clients for domestic violence counseling D. It implied that counselors must break confidentiality to warn intended victims of harm

D

In successful groups, what are the three distinct sections of each group counseling session? A. Confinement, sharing, and closure B. Basic, advanced, and termination C. Self, others, and group D. Warm-up, action, and closure

D

In which of the following areas is specialty counselor certification not yet possible? A. Mental health B. School C. Addictions D. Career

D

Mindfulness is a concept that many counselors integrate into their work with clients. What is the main purpose of mindfulness? A. To provide the client a narrative for his or her experiences B. To rid the client of unhealthy thought patterns C. To provide the client with coping skills in order to avoid self-harm D. To change the client's relationship with their negative thoughts

D

On an inpatient psychiatric unit, patients earn points throughout the day for participating in activities and engaging in treatment. Each day in the afternoon, patients are allowed to "spend" their points on items such as toiletries and snacks in the canteen. What type of reinforcement system is being used? A. Fixed ratio B. Process oriented C. Negative reinforcement D. Token economy

D

Over the past several decades there has been increasing evidence that there are strong relationships between the psyche, nutrition, medications, and body functions. What is the main reason counselors should acknowledge this connection? A. Counselors who understand this connection better typically earn more money and attract more clients B. Insurance companies are requiring more intensive explorations of all aspects of clients' well-being C. Many counselors are returning to school to become psychiatrists or dieticians D. Some behavior is likely rooted in biological and brain functions

D

Reinforcement theory and cognitive information processing are important concepts in whose theory of career counseling? A. Axelrad B. Gottfredson C. Holland D. Krumboltz

D

Statistical regression is BEST explained by which of the following A. Test scores can often predict future scores on related material B. Errors are often made when scoring tests. C. When large numbers of people take a test, it is less likely for individual scores to be very high or very low D. If an individuals score is very low or very high on a pretest, the individuals score will be closer to the mean on the posttest.

D

The General Education Development (GED) test is what type of assessment? A. Ability B. Personality C. Cognitive D. Achievement

D

Theorists from the actuarial perspective believe that the structure of the individual is the foundation from which career development occurs. What are two examples of actuarial theories? A. Problem-solving and needs-based B. Self-directed and conventional C. Life-span and self-concept D. Trait-factor and needs-based

D

Two hundred college students are enrolled in a genetic biology course. Half comprise a control group, while the other half are randomly assigned to participate in an hour-long interactive seminar each week. Students' final test grades are examined at the end of the semester to determine whether the seminar was associated with better comprehension and understanding of course concepts. What type of research design was used? A. Quasi-experimental B. Non-experimental C. Comparative D. True experimental

D

Two tests are administered to subjects participating in a research study. Each test measures true variance and error variance. The correlation between the two tests is .70. What is the amount of true variance measured in common? A. 54% B. 55% C. 70% D. 49%

D

What is a correlation between two variables? A. A variability index B. A reliability index C. A multivariate correlation D. A bivariate correlation

D

What is a goal that group members of a person-centered group may have? A. Become free of scripts and games. B. Increase self-acceptance and move past self-defeating behaviors C. Take responsibility for one's life D. Increase self-understanding and explore a full range of feelings.

D

What is not a kind of analysis of variance (ANOVA)? A. Factorial B. Multivariate C. One-way D. T-test

D

What is the BEST definition of attribution theory? A. The belief that the future is determined more by chance than by being contingent on certain evens B. The expectations that people have for themselves are often higher than what they are capable of C. The belief that rewards are contingent on our own actions D. The explanations people offer for events influence future expectations for success and failure.

D

What is the goal of an inferential approach to statistical analysis? A. To present data that is nonparametric B. To summarize conclusions of a study C. To describe the data collected for a research sample D. To determine the probability of a certain event occurring

D

What is the goal of consultation? A. To triage clients whose needs may be better met elsewhere B. To decrease the possibility of ethical dilemmas C. To strengthen the counselor's documentation skills D. To increase the counselor's ability to handle problems in the future

D

What is the importance of neurotransmitters in the context of counseling individuals with mood disorders? A. Neurotransmitters are solely responsible for memory, so clients can remember counseling sessions and apply concepts in between sessions B. Neurotransmitters are a good predictor of how good someone's prognosis will be after receiving counseling C. Neurotransmitters act as sponges to absorb the extreme emotional reactions that tend to come up in counseling sessions D. Neurotransmitters directly affect a person's mood as well as their emotional and cognitive functioning

D

What is the main purpose of program accountability? A. To deny counselors the ability to take advantage of clients' finances B. To provide regular supervision of counselors new to the field C. To increase counselors' knowledge of a particular skill D. To justify counselors' activities to populations served and to sources of funding

D

When neither the researcher nor the study subjects know which group is the control group and which is the experimental group, this is known as what technique? A. Cross-sectional B. Single-subject C. Halo effect D. Double-blind

D

Which of the following did Jacob Moreno contribute to the field of group psychotherapy? A. The concept of different levels of leadership functions B. Various means of achieving group cohesion C. The importance of altruism to the group dynamic D. The psychodrama technique

D

Which of the following facts about teenage suicide is accurate? A. Suicide rates have climbed drastically in recent years B. Suicide is the fifth leading cause of death among ten to 24-year-olds C. Girls commit about 80 percent of all suicides D. American Indian and Alaska Native teens are more at risk than White or African-American teens

D

Which of the following has been the fastest-growing area of counseling over the past several years? A. Group psychotherapy B. Couples' counseling C. Eye Movement Desensitization and Reprocessing (EMDR) D. Disaster mental health

D

Which of the following is an example of an emic worldview? A. A counselor is directive in her approach with multicultural clients, believing that it is better for them to assimilate rather than attempt to preserve their own beliefs B. A counselor attempts to help all clients, no matter their ethnicity or culture, because she has a global view of humanity C. A counselor believes that all people are more similar than different D. A counselor works closely with clients from a culture different from hers in an attempt to understand their perspectives

D

Which of the following is false about the concept of prejudice? A. It may be positive or negative B. It is a preconceived judgment or opinion C. It can be directed at individuals or groups D. It is defined as intentionally malicious

D

Which of the following is not a 2016 CACREP standard? A. College counseling and student affairs B. Career counseling C.Clinical rehabilitation counseling D. Geriatric counseling

D

Which of the following is seen as an alternative to CACREP? A. AACD B. APGA C. ACA D. MPCAC

D

Which of the following is the best description of systematic eclecticism? A. An approach meant to alleviate anxiety by repeatedly exposing the client to the feared situation B. A way of understanding how individual family members feed off of each other to create dysfunctional dynamics and communication patterns C. A research technique in which researchers choose participants blindly D. An approach to counseling that acknowledges the importance of feelings, thoughts, behaviors, cultural experiences, and societal influences, then applies those that are most relevant to the client(s)

D

Which of the following is the best description of the goal of Bowenian family counseling? A. To better organize the family system and various members' roles B. To solve a specific family problem within a short amount of time C. To increase distance between family members and strengthen boundaries D. To improve dysfunctional patterns that exist in family systems and are often passed down from one generation to the next

D

Which of the following is true about the Affordable Care Act of 2010? A. It allows insurance companies to discriminate based on preexisting conditions B. It gives states the power to decide whether to expand public and private insurance coverage C. It outlawed mandates and subsidies for health insurance D. It requires that mental health care services be treated the same as regular health care

D

Who developed the concept of the life-career rainbow? A. John Krumboltz B. John Crites C. John Holland D. Donald Super

D

Who is known for his belief that developmental tasks, which are the skills a person obtains through maturation, must be accomplished at each stage of growth before the next stage can successfully occur? A. John Watson B. Abraham Maslow C. Aaron Beck D. Robert Havighurst

D

Why is it difficult for counselors to establish a collective identity? A. The counseling profession is declining in popularity B. Counselors lack a single unifying licensing agency C. The counseling profession lacks training standards and common values D. Counselors have a wide range of interests and areas of competence

D

Why is mediation usually recommended for couples going through a divorce? A. It lessens the long-term impacts of divorce on children B. It is a good option for couples who are physically violent toward each other C. It is easier than going through the courts when one of the spouses is missing D. It presents solutions that may not be acceptable unless they are presented by a third party

D

The question below is an example of what type of scale? "On the following scale, how would you describe yourself as a child?" Outgoing ____ ____ ____ ____ ____ Shy Creative ____ ____ ____ ____ ____ Practical Relaxed ____ ____ ____ ____ ____ Anxious A. Likert B. Sociometric C. Unobtrusive D. Semantic Differential

D A semantic differential scale is one way researchers collect information about individuals' opinions and beliefs.

Stanley Strong viewed counseling through the lens of the social influence model. According to Strong, the client generally views the counselor as having all of the following characteristics except which one? A. Attractiveness B. Trustworthiness C. Expertise D. Authoritativeness

D According to Strong's social influence model of counseling, the client may view the counselor as being the expert, in that the counselor has a special set of skills and experience; attractive, in that the client wants to gain the counselor's approval; and trustworthy, in that the counselor is seen as caring and wanting to help.

According to Murray Bowen, what is the basic building block of the family emotional system? A. The mother and father B. The individual C. Childhood experiences D. The triangle

D Bowen's approach is transgenerational, meaning that he believes communication patterns and dynamics are passed down from one generation to the next. He believes that triangles within family systems represent the most basic building blocks of a family's emotional system. In a triangle, emotional discomfort that exists between two family members may be somewhat alleviated by bringing in a third family member to resolve the stress. Bowen also stresses the importance of self-differentiation, projection onto children in families, birth order and sibling position, and societal regression.

In what situation would Caplan's consultation model be most appropriate? A. A counselor asks to meet with her supervisor because she has recently been feeling inadequate at work B. A case manager at a social services agency consults with a coworker in an effort to improve time management skills C. A counselor searches the internet to increase understanding of various types of defense mechanisms D. A counselor treating an individual with an eating disorder consults with another counselor with more experience to discuss the need for family therapy in the case

D Caplan's model of consultation in counseling is a mental health consultation model. In Caplan's model, two professionals discuss issues specific to mental health diagnoses, such as eating disorders, and treatment. The center of discussion can be an individual client or family, the consultee and the client, treatment or a specific program, or the consultee and administration.

Which of the following is an example of cultural encapsulation? A. A person whose primary language is Spanish denies that her ancestors are from another country B. A recent immigrant to the United States begins listening to music by American artists and eating at fast food restaurants C. The president of a successful company aims to hire a culturally diverse staff D. An individual who identifies as Christian believes anyone who argues with his theology is wrong and uncivilized

D Cultural encapsulation occurs when an individual holds a narrow viewpoint and refuses to take other perspectives or viewpoints into consideration.

Which of the following helps to maintain homeostasis within a family system? A. Interdependence B. Positive feedback loops C. Equifinality D. Negative feedback loops

D Cybernetics is the study of methods of feedback control, both negative and positive. Negative feedback loops are patterns of interaction that maintain stability and homeostasis within the family system. Positive feedback loops are patterns of interaction that can facilitate change. Equifinality refers to different individuals reaching the same goal through different methods or routes. Interdependence simply refers to the fact that family members rely on and are influenced by each other.

Who developed the law that states the consequences of a behavior determine the strength of the response connection? A. John Watson B. B.F. Skinner C. Robert Havighurst D. Edward Thorndike

D Edward Thorndike developed the law of effect, which states that when a reward follows a stimulus-response connection, the connection is strengthened

Which of the following is the best example of an encore career? A. Jesse went to school to become an actor, but after graduation, he decides to pursue a career in psychology B. At age 40, Karen, a psychiatrist, decides to make a career change and become a pharmacist C. Kevin is employed full-time as an architect, but on the weekends takes photographs at weddings and other occasions D. After 30 years working for the federal government, Sam decides to take a job at a local home improvement store

D Encore careers refer to those times when, for different reasons, retired workers return to work.

Crystallized intelligence refers to what type of abilities? A. The ability to verbalize emotional experiences B. The tendency to convert failures into new opportunities C. The ability to solve new problems quickly D. Acquired and learned skills

D Fluid intelligence refers to the ability to solve new problems quickly and is independent of education and culture. Crystallized intelligence, on the other hand, is comprised of acquired skills and can be influenced by culture, personality, and education.

In group counseling, what is the primary purpose of linking? A. To develop insight into principles of paraverbal communication B. To encourage group members to think critically about their own role in the problem C. To stop unproductive behaviors in the group D. To facilitate members helping each other with their problems

D Linking is the process of looking for common themes in the group content or process and connecting them. This approach can help members work on each other's problems.

Counselors and other mental health professionals have many different ways of viewing human growth and development. These include qualitative or quantitative, continuous or discontinuous, and mechanistic or organismic. Which of the following is an example of organismic development? A. Language development B. Sexual development C. Reflexive behavior D. Ethical development

D Organismic development is the belief that the individual (or other organism) is involved in the development. This can lead to change or discontinuity depending on the individual. Ethical development and moral development are both regarded as organismic development. Reflexive behavior is mechanistic development, the reduction of behavior to common elements. Language development is discontinuous development, and sexual development is qualitative development.

Which of the following is the best example of a self-help group? A. A group for individuals with a history of alcohol abuse, led by a psychotherapist B. A group for parents who have lost a child, led by a professional counselor C. A group led by a school counselor for parents of children in special education settings D. A group for survivors of sexual abuse, led by a survivor

D Self-help groups are support systems to help with psychological stress of some sort and are typically not led by a professional

What is one advantage of using interval data? A. Nonparametric statistics can be used B. Data can give researchers an idea of why differences in scores occur C. Numbers can be compared by ratios D. There is a constant unit of measurement

D There are four levels of measurement: nominal, ordinal, interval, and ratio. With interval data, the numbers on a scale have the same amount of the variable throughout the scale; for instance, degrees on the Fahrenheit temperature scale.

If the correlation between two tests is .80, what is the coefficient of nondetermination? A. 10% B. 64% C. 20% D. 36%

D To find the coefficient of nondetermination, first find the coefficient of determination by squaring the correlation (.80 x .80 = .64, or 64%). Then subtract this from 100% to find the coefficient of nondetermination (100% - 64% = 36%).

Two tests are administered to subjects participating in a research study. Each test measures true variance and error variance. The correlation between the two tests is .70. What is the amount of true variance measured in common? A. 54% B. 55% C. 70% D. 49%

D True variance is measured by finding the square of the correlation. In this question, the correlation between the two tests is .70. Since .70 x .70 = .49, the true variance between the two tests is 49%.

Donald Super is highly regarded for his developmental approach to career counseling. All of the following are vocational development stages described by Super except which one? A. Decline B. Establishment C. Maintenance D. Crystallization

D Vocational development stages are growth (development of capacity and interests), exploratory (tentative choices made), establishment (trial and stabilization), maintenance (adjustment process), and decline (retirement). The vocational development tasks are crystallization (forming a vocational goal), specification (moving to a more specific career choice), implementation (entering employment), stabilization (performing a job), and consolidation (achieving status and advancing).

Which of the following is often most appealing to multicultural individuals in Transactional Analysis (TA) groups? A. TA gives a broad understanding of human nature B. TA groups are action-oriented C. All TA concepts are easy for multicultural clients to understand D. The structure of TA groups, and contracts that can be adjusted for different cultural values

D Transactional Analysis (TA) groups focus on a combination of action and affect, and the leader is regarded as the diagnostician and teacher


संबंधित स्टडी सेट्स

NU140- Chapter 27 Safety, Security, and Emergency Preparedness

View Set

Synonyms and Antonyms 10.2_changed

View Set

FINAL EXAM FINA 365 (Chpt. 26,27,29)

View Set

Ch 11 D&W+ Skill Building: Diabetes Exchanges

View Set

"The Notorious Jumping Frog of Calaveras County" Test

View Set

Chapter 1 - Introduction to Computers, the Internet and Java

View Set